Vous êtes sur la page 1sur 18

www.academiamir.

com Comentarios test de clase A Mir

COMENTARIOS desde las 28 semanas


4. El reflejo de la marcha aparece al rozar los pies con una superficie
TEST DE CLASE dura y está presente desde el nacimiento
5. El reflejo de paracaídas no desaparece nunca
PEDIATRÍA
Esta pregunta se contesta recordando en qué momento desaparecen los refle-
jos arcaicos y prestando especial atención a que es verdad que el reflejo de suc-
FASE DE CONTACTO ción está presente desde la 28 semanas, pero NO está coordinado con la deglu-
ción hasta la semana 32. El resto de las respuestas son correctas:
1. ¿Cuál es la causa más frecuente de mortalidad en niños menores de un año? Reflejo de Moro: desaparece 4º-6º mes. Consiste en un movimiento de aper-
tura y posterior cierre de los brazos (abrazo) en respuesta a una hiperextesión
1. Neoplasias de la cabeza.
2. Accidentes Reflejo de prensión palmar: desaparece 4º-6º mes. Presionamos en la mano
del niño con nuestro dedo y este cierra la mano y nos agarra el dedo.
3. Procesos perinatales Reflejo de prensión plantar: desaparece 9-10 meses. Presionamos con nues-
4. Homicidios tro dedo en la planta del pie del recién nacido y observamos que flexiona los
5. Infecciones dedos del pie.
Reflejo de succión: desaparece 4º mes
Esta es una pregunta directa que se responde únicamente recordando cuáles Presente desde las 28 semanas de edad gestacional
son las causas más frecuentes de mortalidad: Coordinado con deglución desde las 32 semanas de edad gestacional
Feto → Insuficiencia placentaria Reflejo de Galant o incurvación del tronco: desaparece 4º-6º mes.
RNPT → Inmadurez Estimulamos al niño en un lateral de la espalda y el niño incurva el tronco
RNT → Alteraciones congénitas hacia ese lado.
< de 1 año → Procesos perinatales Reflejo del paracaídas: no desaparece nunca. Es el que debe hacer que pon-
1 - 14 años → Accidentes gamos las manos en el suelo (¡¡¡y no la nariz!!!) cuando nos caemos.
Reflejo de la marcha: presente desde el nacimiento. Ponemos al niño sobre
2. ¿Cuál de las siguientes características debe ser considerada patológica en un una superficie dura y observamos que hace intención de dar algún paso.
recién nacido?
5. Acude a la urgencia de su hospital un recién nacido dado de alta de la mater-
1. Pérdida del 8% del peso de recién nacido en la primera semana de nidad hace 48 horas. Los padres refieren llora continuamente y que no moja los
vida pañales. El PRN era de 3.250 g y actualmente pesa 2.850 g. En la exploración
2. Eritema tóxico alérgico se observa una fontanela algo deprimida y una temperatura axilar de 37,8º C.
3. Edema localizado ¿Cuál sería su actitud?
4. Craneotabes parietal
5. Cutis marmorata 1. Realizar analítica completa, observación y probable ingreso hospi-
talario
Esta pregunta se refiere a las características normales del recién nacido. Debéis 2. Rehidratar urgentemente con suero hipernatrémico
recordar que hasta el 10% de pérdida de peso es fisiológica en los primeros 3. Lo más probable es que se deba a altas temperaturas ambientales
días de vida, recuperando el peso del nacimiento o superándolo a las 2 sema- y no es preciso tomar ninguna medida
nas de vida. El eritema tóxico alérgico es un exantema normal en el recién naci- 4. Dado que el paciente no tiene fiebre, es muy raro que se deba a
do que aparece al segundo o tercer día de vida, consiste en pápulas blanque- una infección, por lo que se debe tranquilizar a los padres ante la
cinas sobre una base eritematosa, se relacionan con la lactancia materna y des- benignidad del proceso
aparecen espontáneamente. El recién nacido puede presentar edema localiza-
5. Se debe solicitar sangre urgente porque lo más probable es que
do, por ejemplo en las zonas de presentación, sin que esto guarde ninguna
relación con el edema generalizado de patologías mucho más graves (por haya que trasfundir al paciente
ejemplo el hidrops fetal). El craneotabes parietal se considera normal en el
recién nacido y habría que diferenciarlo del craneotabes patológico (occipital, Esta pregunta expone un caso clínico, lo primero es hacerse una idea del cua-
temporal o si persiste más allá del periodo neonatal). La respuesta correcta en dro que sospechamos que tiene nuestro paciente y luego encontrar la actitud
esta pregunta es cutis marmorata, que se puede ver en algunos recién nacidos más adecuada. Cuando un recién nacido llora continuamente hay que pensar
cuando pierden temperatura (inmediatamente después del parto) pero que en que tenga hambre, dolor o bien algún cuadro que le produzca irritabilidad. En
general debe considerarse un signo de mala perfusión periférica y, por tanto, este caso nos cuentan que el niño no moja los pañales, signo que hay que
de gravedad. interpretar como falta de líquido en el organismo (deshidratación). También
nos dicen que el PRN era de 3.250 g y que actualmente pesa 2.850 g, con lo
que se observa que ha perdido el 12% de su peso. Además la fontanela está
3. Un recién nacido presenta a los pocos días de vida una masa palpable y de algo deprimida (signo también de deshidratación) y tiene febrícula (que no
movimiento transversal en la zona lateral del cuello, sobre la patología que sos- siempre es sinónimo de infección y que también puede ser un signo de deshi-
pecha ¿Cuál de las siguientes afirmaciones le parece INCORRECTA? dratación). Es fundamental recordar que a todo recién nacido que acuda con
algún signo que no se considere fisiológico se le debe hacer un pequeño estu-
1. Probablemente el parto fue traumático dio para descartar complicaciones, ya que estas son frecuentes y peligrosas en
2. Lo más frecuente es que se trate de un hematoma el recién nacido. Por tanto, el primer paso sería un análisis de sangre y según
3. Puede evolucionar hacia la calcificación el resultado decidir el tipo de rehidratación.
4. Si existen dudas diagnósticas se debería realizar una ecografía cer-
vical 6. La causa más frecuente de masa abdominal en el recién nacido es:
5. El tratamiento es la extirpación quirúrgica
1. Íleo meconial
El cuadro clínico de esta pregunta corresponde a un hematoma del esterno- 2. Trombosis de la vena renal
cleidomastoideo, que probablemente fue consecuencia de un parto traumáti- 3. Nefroblastoma
co. Lo normal es que la zona del hematoma evolucione hacia la calcificación 4. Hidronefrosis
espontánea y el diagnóstico se haría con una ecografía del músculo visualizan- 5. Neuroblastoma
do el aumento de la densidad de la sangre del hematoma o la propia calcifica-
ción. El tratamiento NO sería en ningún caso la cirugía, sino la rehabilitación y Esta es una pregunta directa simplemente para que recordéis que la causa más
la fisioterapia. El diagnóstico diferencial podría plantearse con un tortícolis con-
frecuente de masa abdominal en el recién nacido es la hidronefrosis.
génito, en este caso relacionado con un moldeamiento intraútero, mucho más
infrecuente que el hematoma.
7. La hernia diafragmática congénita:
4. Con respecto a los reflejos arcaicos, señale cual de las siguientes afirmacio-
nes es incorrecta: 1. Es una malformación congénita del diafragma, del árbol bronquial
y del arco aórtico
1. El reflejo de Moro consiste en un movimiento de abrazo cuando se 2. La malformación digestiva más frecuentemente asociada es la mal-
extiende de forma brusca la cabeza rotación intestinal
2. El reflejo de prensión palmar desaparece al 4º-6º mes y el de pren- 3. El tratamiento quirúrgico debe realizarse en las primeras 24 horas
sión plantar a los 9-10 meses de vida
3. El reflejo de succión coordinado con la deglución está presente 4. Se asocia frecuentemente con una comunicación interauricular

1 ] PEDIATRÍA [
www.academiamir.com Comentarios test de clase A Mir

5. La incidencia es más elevada en el lado derecho Es una pregunta directa, sin mucha relevancia para el MIR, simplemente sirve
para destacar que actualmente pueden sobrevivir niños incluso de 500 gramos.
Esta pregunta pertenece a un examen MIR y para responderla hay que cono- La clasificación sería la siguiente:
cer las principales características de las hernias diafragmáticas congénitas. Con Recién nacido de extremadamente bajo peso (RNEBP): •≤ 1000 g
respecto a la primera opción hay que recordar que la hernia diafragmática no Recién nacido de muy bajo peso (RNMBP): ≤ 1500 g
suele acompañarse de otras malformaciones en el árbol bronquial, ni en el arco Recién nacido de bajo peso (RNBP): > 1500 g
aórtico ni en las aurículas. Por el contrario, dado que generalmente el intestino Recién nacido de peso excesivo (RNPE): > 4 Kg
en desarrollo entra en la cavidad torácica, es frecuente que exista malrotación
intestinal (la malformación digestiva más frecuentemente asociada es la malro- 11. ¿Cúal de los siguientes datos NO se tiene en cuenta en la prueba de Apgar?
tación intestinal). Lo más frecuente es que se produzcan en el lado izquierdo,
parece que el hígado tiene un cierto efecto protector. El tratamiento quirúrgi- 1. Color
co debe realizarse lo antes posible en las hernias de Bochdalek, pero en las
2. Reflejos
Morgagni (que generalmente son asintomáticas) no es imprescindible interve-
nir con tanta velocidad.
3. Tono
4. Tensión arterial
8. Un recién nacido de raza negra presenta una tumefacción periumbilical recu- 5. Frecuencia cardiaca
bierta por piel. Señale la afirmación incorrecta de las siguientes:
Esta pregunta pertenece a un examen MIR y con ella se pretende que recuer-
des cuales son lo datos que se tienen en cuenta al hacer una valoración de un
1. Se debe hacer diagnóstico diferencial con el onfalocele recién nacido según el test de Apgar. Es suficiente con recordar el siguiente
2. La tumefacción aumenta con el llanto, la tos o la defecación cuadro:
3. Es raro que se estrangule
4. Con frecuencia desaparece espontáneamente al año de vida
5. Es una protrusión de epiplon o intestino delgado 0 1 2

Lo que nos describen en este caso es una hernia umbilical que, a diferencia del
onfalocele, está cubierta por piel y no ofrece lugar a duda sobre su diagnósti- FRECUENCIA
co. Consiste en una protrusión de epiplon o intestino delgado que suele Ausentes < 100 >100
CARDIACA
aumentar con el llanto, la tos o la defecación. Dado que el orificio herniario no
es demasiado estrecho, es muy raro que se estrangule. Lo normal es que des-
ESFUERZO
aparezca espontáneamente al año de vida y es muy raro que precise cirugía Ausente Pobre, irregular Llanto vigoroso
RESPIRATORIO
para su resolución.

9. Un recién nacido de 24 días de vida, que desprendió el cordón umbilical a los TONO MUSCULAR Laxo Cierta flexión Activo
8 días y cuyo ombligo no ha cicatrizado, presenta una tumoración umbilical
roja, prominente, circular y que segrega un contenido alcalino. Los padres a
RESPUESTA A
veces notan en ella un ruido de gases. El diagnóstico más probable es: Ausente Gesticula Tos, estornudo
ESTÍMULOS

1. Persistencia del conducto onfalomesentérico


2. Fístula vésico-umbilical COLOR Azul pálido Acrocianosis Rosado
3. Granuloma umbilical
4. Gastrosquisis
5. Persistencia del uraco
12. Un recién nacido presenta en el primer minuto de vida una frecuencia car-
Esta pregunta pertenece a un examen MIR y en ella nos cuentan patología del diaca de 90 lpm, su esfuerzo respiratorio es nulo, tiene una postura en ligera
cordón umbilical y tenemos que ser capaces de reconocer la patología a la que flexión con cierto grado de acrocianosis y al introducirle la SNG hace algún
se refieren con las características que nos dan. En general debemos fijarnos en gesto facial. ¿Cuál sería su puntuación en el test de Apgar?
las características de las secreciones (especialmente del pH). Así, el hecho de
que el pH de la secreción sea alcalino hace sospechar que se trate de la persis-
1. 4
tencia del conducto onfalomesentérico.
2. 3
3. 2
PERSISTENCIA DEL 4. 1
PERSISTENCIA DEL GRANULOMA
CONDUCTO 5. 0
URACO UMBILICAL
ONFALOMESENTÉRICO
Esta pregunta también hacer referencia al test de Apgar y lo que te solicitan en
Conducto atlantoideo
ella es que des la puntuación al recién nacido en función de unos signos clíni-
cos. Así, teniendo en cuenta el cuadro de la respuesta anterior, una frecuencia
Blando cardiaca de 90 lpm suma 1 punto, un esfuerzo respiratorio nulo suma 0 pun-
Fístulas tos, la postura en ligera flexión suma 1 punto, la acrocianosis otro punto y si al
Duro Blando, granular, vascu-
Prolapsos introducirle la SNG hace algún gesto facial se suma otro punto. La puntuación
Pólipo rojo larizado
Pólipos
global es de 4 puntos.
Meckel
Mucosa urinaria o 13. En la asistencia al recién nacido, señale cual de las siguientes afirmaciones
digestiva es correcta:
Líquido amarillo
Mucoso 1. La primera medida a tomar es aspirar la cavidad oral
Orina Secreción
pH alcalino
pH ácido mucopurulenta
Gases
2. Se debe administrar a todos los recién nacidos profilaxis ocular con
Heces nitrato de plata
3. Se debe administrar vitamina K a aquellos recién nacidos cuya
ECO madre presente trastornos de coagulación durante el parto
4. Para evitar la hipotermia inmediatamente posparto se aplican al
Resección Nitrato de Ag y alcohol Resección recién nacido paños húmedos calientes
5. Si el líquido amniótico está teñido de meconio, no se debe esti-
mular la respiración espontánea del recién nacido hasta haber aspi-
10. Se considera recién nacido de muy bajo peso: rado orofaringe y tráquea

1. <500 g En esta pregunta se comentan algunas de las primeras actuaciones médicas


2. <1000 g con un recién nacido y nos preguntan cuál de todas es correcta. Cuando asis-
timos a un recién nacido, la primera medida a tomar es secarle, calentarle,
3. <1500 g
abrirle la vía aérea y aspirarle la cavidad oral, igual que en el ABC de la reani-
4. <1750 g mación cardiopulmonar (comprobar nivel de conciencia, dar estímulos, com-
5. <2000 g probar ausencia de respiración e iniciar RCP comenzando por abrir la vía aérea).

2 ] PEDIATRÍA [
www.academiamir.com Comentarios test de clase A Mir

Es cierto que a todos los recién nacidos se les debe administrar profilaxis ocu- 16. ¿Cuál de las siguientes asociaciones NO es correcta?
lar frente a las infecciones producidas por gérmenes del canal del parto, pero
esta profilaxis NO se hace con nitrato de plata (técnica antigua), sino con 1. Ductus / ligamento arterioso
pomadas antibióticas de tetraciclinas o eritromicina. También es cierto que se
2. Conducto de Arancio / ligamento umbilical medial
les debe administrar vitamina K en las primeras horas tras el parto, pero a
TODOS los recién nacidos y NO sólo a aquellos cuya madre presente trastornos 3. Arterias umbilicales / ligamento umbilical lateral
de coagulación durante el parto. Para evitar la hipotermia inmediatamente pos- 4. Vena umbilical / ligamento redondo
parto se aplican al recién nacido paños calientes, pero estos deben estar secos 5. Foramen oval / tabique interauricular
y NO húmedos. Ya se ha comentado previamente que la primera medida a
tomar en la reanimación de un recién nacido es estimularle, secarle, calentarle, Esta es una pregunta directa donde lo fundamental es recordar a qué estruc-
abrirle la vía aérea y aspirarle secreciones, pero existe una excepción: las aguas turas da lugar cada uno de los conductos que están presentes en el feto. Así,
meconiales. Si nos cuentan que el líquido está teñido de meconio, no se debe las estructuras en la circulación fetal y sus respectivos homólogos en la circula-
estimular la respiración espontánea del recién nacido hasta haber aspirado oro- ción adulta son las siguientes:
faringe y tráquea, ya que si inicia inspiraciones espontáneas puede estar intro- Ductus arterioso → ligamento arterioso
duciendo el meconio hasta sus alvéolos y provocando así un síndrome de aspi- Conducto de Arancio → ligamento venoso
ración meconial. Arterias umbilicales → ligamento umbilical lateral
Vena umbilical → ligamento redondo
14. Acude usted a paritorio a reanimar a un recién nacido y se encuentra con Foramen oval → tabique interauricular
que la frecuencia cardiaca es de 60 lpm, el esfuerzo respiratorio está ausente,
está absolutamente hipotónico y de color azul pálido y la respuesta a estímulos 17. Con respecto a los parámetros hematológicos del recién nacido, señale la
es nula. ¿Cuál sería su actitud inicial? opción correcta:

1. Ventilación con mascarilla y PPI y O2 durante 30 segundos y, si no 1. Dado que durante el parto se produce un sangrado importante, lo
hay respuesta, intubación endotraqueal normal es que el recién nacido presente Hb de 9-10 mg/dl con un
2. Intubación y masaje cardiaco hematocrito de 27-30% que recuperará espontáneamente en los pri-
3. Secar, calentar, estimulación táctil y abrir y aspirar vía aérea meros días de vida
4. Adrenalina, bicarbonato y masaje cardiaco 2. Entre los 3 y los 6 meses aparece una anemia fisiológica que cursa
5. Intubación endotraqueal y expansión con SSF con Hb de entre 9,5 y 14 mg/dl y con consumo de los depósitos de
ferritina
En esta pregunta nos sitúan en el paritorio ante un recién nacido con impor- 3. La hemoglobina fetal desaparece en los 2 primeros meses de vida
tante afectación del estado general (frecuencia cardiaca es de 60 lpm, el y su composición se asemeja a la del adulto
esfuerzo respiratorio está ausente, está absolutamente hipotónico y de color 4. La Hb fetal se compone un 50% de HbF y el otro 50% de Hb A1
azul pálido y la respuesta a estímulos es nula, corresponde a un Apgar de 1). y A2
Una vez superado el susto inicial se debe actuar igual que en todos los casos,
5. Cifras de leucocitos superiores a 10.000/mm3 en las primeras
es decir, seguir el ABC. Así, la primera medida a tomar es secar, calentar, esti-
mulación táctil y abrir y aspirar vía aérea. horas de vida deben hacer sospechar la existencia de una infección

15. Un recién nacido queda expuesto a temperatura ambiente durante más En esta pregunta se pretende destacar que existen ciertas diferencias entre la
sangre del recién nacido y la de los niños mayores o adultos y nos preguntan
tiempo del debido y a los pocos minutos comienza con dificultad respiratoria cuál de las opciones que nos dan es correcta. Con respecto a la primera de
observándose que no existe disociación toraco-abdominal pero que para respi- ellas, no es cierto que el sangrado que se produzca durante el parto (funda-
rar mantiene cierto grado de tiraje intercostal, con discreta retracción xifoidea mentalmente de origen materno) afecte de forma tan severa a un recién naci-
y leve aleteo nasal. En la auscultación sólo se aprecia un suave quejido. ¿Cuál do, donde lo esperable es encontrar tendencia a la policitemia (hemoglobina
sería su puntuación en el test de Silverman? 17 – 19 g/dl y hematocrito 45 – 60%). Sin embargo, lo esperable es que esta
situación de plenitud dure poco y que entre los 3 y los 6 meses aparezca una
1. 0 anemia fisiológica que cursa con Hb de entre 9,5 y 14 mg/dl y con consumo de
los depósitos de ferritina. La hemoglobina fetal NO desaparece en los 2 prime-
2. 1
ros meses de vida y su composición no se asemeja a la del adulto hasta los 6 –
3. 2 12 meses, se compone un 70% de HbF, un 29% de Hb A1 y un 1% de A2. El
4. 3 parto es una situación de estrés que puede hacer que exista una leucocitosis en
5. 4 el recién nacido de hasta 25.000/mm3.

Los recién nacidos son muy vulnerables, especialmente a cambios de tempera- 18. En el líquido amniótico de un feto afecto de síndrome de Down ¿qué patrón
tura. No debemos olvidar que durante el parto están expuestos a la tempera- bioquímico esperaría encontrar?
tura materna pero, una vez que nacen, su temperatura es la del ambiente, lo
que hace que el recién nacido pierda muchos grados de temperatura de forma
muy rápida. Esta situación se manifiesta en ellos como un distrés respiratorio 1. α-fetoproteína ↓, estriol ↓, proteína A ↑, hCG ↑ e inhibina ↑
que se puede clasificar según su gravedad en función del test de Silverman. 2. α-fetoproteína ↓, estriol ↓, proteína A ↓, hCG ↓ e inhibina ↓
3. α-fetoproteína ↑, estriol ↑, proteína A ↑, hCG ↑ e inhibina ↑
4. α-fetoproteína ↑, estriol ↑, proteína A ↓, hCG ↓ e inhibina ↓
5. α-fetoproteína ↓, estriol ↓, proteína A ↓, hCG ↓ e inhibina ↑
0 1 2
En esta pregunta se hace referencia a la situación hormonal del líquido amnió-
Torax fijo Respiración y tico y a las modificaciones que este puede sufrir según la patología del feto.
DISOCIACIÓN Normal Sólo es interesante recordar que los defectos del tubo neural tienen niveles de
Mueve abdomen balanceo
α-fetoproteína elevados y que en el caso de un feto afecto de síndrome de
Down encontraríamos niveles de α-fetoproteína disminuidos junto con aumen-
Audible con Audible to de ßhCG (“el embarazo de un síndrome de Down es mucho embarazo”).
QUEJIDO Ausente
fonendo sin fonendo Prestar especial atención a las características del líquido amniótico de un feto
afecto de síndrome de Down: α-fetoproteína ↓, estriol ↓, proteína A ↑, ßhCG ↑
+ supra e e inhibina ↑.
TIRAJE Ausente Intercostal
infraesternal
19. ¿Cuál de las siguientes NO suele ser una complicación típica de un recién
ALETEO Ausente Discreto Intenso nacido pretérmino?

1. Síndrome de distrés respiratorio


RETRACCIÓN 2. Distocia de hombros durante el parto
Ausente Discreto Intenso
XIFOIDEA 3. Enterocolitis necrotizante
4. Hipoglucemia
5. Hemorragia intraventricular
Que no exista disociación toraco-abdominal suma 0 puntos, cierto grado de
tiraje intercostal suma 1 punto, la discreta retracción xifoidea suma otro punto, Esta es una pregunta directa sobre diversas patologías que puede padecer el
el leve aleteo nasal suma otro punto y que el quejido sólo sea audible con recién nacido prematuro, todas se explicarán en preguntas posteriores. La res-
fonendoscopio suma otro punto. La puntuación global es de 4 puntos. puesta en este caso es bastante lógica, es rarísimo que un recién nacido pre-

3 ] PEDIATRÍA [
www.academiamir.com Comentarios test de clase A Mir

maturo sufra una distocia de hombros, dado su pequeño tamaño, el resto de En esta pregunta se deben tener claro cuáles son las características de cada una
los cuadros sí son típicos de RNPT. de las parálisis braquiales y de este modo descartar la que no corresponde a
una parálisis superior o de Duchenne-Erb (la respuesta incorrecta es la 5, ya que
20. Una de las siguientes características NO corresponde a un cefalohematoma: en las parálisis superiores no pueden hacer el reflejo de Moro ni el bicipital pero
mantienen el palmar porque está controlado por raíces inferiores).
1. No alteraciones en la piel
2. Desaparece en semanas o meses 23. Un recién nacido de peso elevado para la edad de gestación, tras un parto
3. Atraviesa suturas traumático, manifiesta a las pocas horas de vida un cuadro de shock con hipo-
4. Tarda horas en aparecer tensión, hipoglucemia e hiponatremia. Sobre la patología que sospecha, seña-
5. No precisa tratamiento le cuál de las siguientes afirmaciones NO es correcta:

Para responder a esta pregunta hay que tener claras cuales son las diferencias 1 . El 90% son unilaterales y el 75% aparecen en el lado derecho
entre el cefalohematoma y el caput sucedaneum. Todas las características de 2. La anoxia y las infecciones fulminantes también pueden provocarla
las respuestas pertenecen a los cefalohematomas salvo que, dado que el san- 3. Pueden existir cambios en la coloración local de la piel o incluso
grado se contiene debajo del periostio y este se ancla en las suturas, lo normal ictericia
es que NO las atraviese. 4. El diagnóstico se realizará mediante TAC
5. Puede estar indicado el tratamiento quirúrgico en situaciones agudas
21. Señale cuál de las siguientes características sobre loas hemorragias intra-
craneales es correcta: En todos los casos clínicos debes actuar igual, lo primero es llegar al diagnósti-
co de la patología que te cuentan, que en este caso se trata de una hemorra-
1. Los hematomas subdurales son típicos de los RNPT y tanto si son gia suprarrenal. Uno de los datos claves es que te cuentan que el recién naci-
asintomáticos como si tienen síntomas deben ser tratados mediante do tiene un peso elevado para la edad de gestación, esto suele favorecer par-
tos traumáticos y que se lesionen vísceras. La clínica es la que describen, una
punción
situación de shock con hipotensión (por la hemorragia), hipoglucemia (porque
2. La hemorragia intraparenquimatosa suele asociarse a situaciones faltan las hormonas contrarreguladoras hiperglucemiantes) e hiponatremia
de hipoxia-isquemia (porque falta la aldosterona encargada de retener Na y eliminar K). De estas
3. El diagnóstico de la hemorragia subaracnoidea se hace funda- hemorragias es cierto que el 90% son unilaterales y el 75% aparecen en el lado
mentalmente mediante TAC derecho debido a que el hígado tiene cierto efecto de “cascanueces” y macha-
4. La hemorragia intraventricular es típica de RNT que sufren trau- ca a la suprarrenal. No sólo los partos traumáticos provocan este cuadro, se
matismos durante el parto, aunque también se deben sospechar puede ver en situaciones de anoxia o de infecciones fulminantes. Dado que la
hemorragia puede ser cuantiosa, pueden existir cambios en la coloración local
malos tratos
de la piel o incluso ictericia. Para el diagnóstico, siempre que se pueda se debe
5. La leucomalacia periventricular es una necrosis de la sustancia evitar irradiar a los niños, y en este caso las glándulas son visibles mediante
blanca periventricular y de las fibras de la capsula interna cuyos pri- ECO abdominal (NO hace falta realizar un TAC). Puede estar indicado el trata-
meros síntomas aparecen a las 24 horas de vida miento quirúrgico en situaciones agudas para intentar controlar la hemorragia.

De las hemorragias intracraneales debes conocer las más importantes, espe- 24. ¿Cuál es la lesión más frecuente en un parto?
cialmente las hemorragias subdurales, subaracnoideas e intraparenquimatosas,
el resto de las hemorragias corresponden a patología del adulto y se estudia-
1. Hematoma subcapsular hepático
ran en otras asignaturas. Observa cuál es el factor de riesgo para cada hemo-
rragia, su clínica, diagnóstico y tratamiento. Así, la primera opción no es correc- 2. Rotura esplénica
ta porque los hematomas subdurales son típicos de RNT y generalmente no 3. Hemorragia suprarrenal
precisan tratamiento. Las hemorragias intraparenquimatosas suelen asociarse a 4. Fractura de clavícula
situaciones de hipoxia-isquemia. El diagnóstico de la hemorragia subaracnoi- 5. Epifisiolisis de húmero
dea se hace fundamentalmente con punción lumbar y no mediante TAC, ya
que no existe una colección que resulte visible con una prueba de imagen y Esta es una pregunta directa para que recordéis que la lesión más frecuente en
precisa punción para ver hematíes en el LCR. La hemorragia intraventricular un parto es la fractura de clavícula.
sólo puede aparecer en RNPT ya que sangran vasos de la matriz germinal sube-
pendimaria que desaparecen a partir de las 28 semanas de edad de gestación.
La leucomalacia periventricular corresponde efectivamente a la descripción que 25. Señale cuál de las siguientes características NO corresponde a la respiración
dan en la pregunta pero la clínica no aparece hasta el final de la lactancia. periódica del RNPT:

22. ¿Cuál de las siguientes características no pertenece a una parálisis braquial 1. Se caracteriza por alternar ritmo regular con episodios de apnea
de Duchenne-Erb? intermitente de pocos segundos de duración, seguidos de respiración
rápida compensadora
1. Es la más frecuente 2. Con cierta frecuencia se asocian coloración cianótica y bradicardias
2. Afecta a las raices C5-C6 transitorias
3. El brazo se muestra en adducción y rotación interna con pronación 3. Cesa de forma espontánea
del antebrazo 4. No tiene importancia pronóstica
4. Puede asociar a alteraciones del nervio frénico (C4) 5. Aumenta durante las fases 3 y 4 de sueño
5. El reflejo de Moro y el bicipital están intactos pero el reflejo palmar
está ausente
PAUSAS DE APNEA RESPIRACIÓN PERIÓDICA
PARÁLISIS
ERB-DUCHENNE DÈJERINE-KLUMPKE 5-10 segundos seguido de
BRAQUIAL 10-20 segundos
respiración rápida compensadora
MECANISMOS DE Distocia de hombros
PRODUCCIÓN Distocia de nalgas
La más frecuente RNPT 1ª semanas RNPT 1os meses
FRECUENCIA C7-C8-(D1)
(C4)-C5-C6
RAICES
Brazo en aducción y Cede 36 semanas EG
rotación interna (postura Mano paralítica
CLÍNICA
en propina de maitre) +/- Síndrome de Horner Estímulo, ventilación con mascarilla y
Cese espontáneo
+/- Dificultad respiratoria O2, CPAP, cafeína, transfusiones, EPO
REFLEJOS:
DE MORO Ausente o asimétrico Normal Cianosis y bradicardia No cambios
BICIPITAL Ausente Normal
PRENSIÓNPALMAR Intacto Ausente
↑ Fase REM ↑Ffases 3 y 4
ASOCIACIÓN Parálisis frénica (C4) D1
Peor pronóstico No importancia pronóstica
PRONÓSTICO Bueno Peor
Inmovilización parcial Inmovilización parcial
TRATAMIENTO intermitente intermitente En esta pregunta lo fundamental es saber diferenciar entre dos cuadros muy
(1-2 semanas) (1-2 semanas) parecidos: la respiración periódica del recién nacido prematuro y las apneas de

4 ] PEDIATRÍA [
www.academiamir.com Comentarios test de clase A Mir

la prematuridad. El diagnóstico diferencial es fácil de realizar según el cuadro descenso de las presiones pulmonares, manteniéndose estas altas
adjunto. como en la vida intrauterina
Así, no es cierto que la respiración periódica del prematuro asocie coloración 2. Los neumotórax suelen cursar con disnea, taquipnea y cianosis
cianótica ni bradicardias transitorias.
generalmente de inicio brusco
3. Tanto el hemotórax como el neumopericardio cursan con tonos
26. Un prematuro de 32 semanas presenta a la hora de vida una disnea pro- cardiacos apagados
gresiva con cianosis y tiraje. En la radiografía hay un patrón de vidrio esmerila- 4. La displasia broncopulmonar es una enfermedad hereditaria auto-
do y broncograma aéreo. A pesar de la ventilación, el oxígeno y los antibióticos sómico recesiva que aparece con más frecuencia en los recién naci-
continúa mal. ¿Qué tratamiento añadiría en primer lugar? dos a término
5. En la enfermedad pulmonar crónica o fibrosis pulmonar intersti-
1. Indometacina oral cial, la principal causa de muerte es la insuficiencia cardiaca y la bron-
2. Ibuprofeno intravenoso quiolitis obliterante
3. Prednisona intravenosa
4. Bicarbonato intravenoso En esta pregunta se hace un breve repaso de otras patologías respiratorias del
5. Surfactante endotraqueal recién nacido, debes conocer algunas características generales sobre ellas aun-
que las más importantes son la enfermedad de membrana hialina y la taquip-
Esta pregunta corresponde a una pregunta de exámenes MIR anteriores y se nea transitoria del recién nacido. Es correcto que la hipertensión pulmonar per-
refiere a patología respiratoria del recién nacido. Dentro de los distintos cua- sistente se produce cuando no hay descenso de las presiones pulmonares,
dros respiratorios lo fundamental es identificar el factor de riesgo para identifi- manteniéndose estas altas como en la vida intrauterina. Los neumotórax sue-
car el diagnóstico, porque clínicamente se manifiestan de forma parecida. En len cursar con disnea, taquipnea y cianosis, generalmente de inicio brusco y
el caso que nos presentan el niño es prematuro, por lo que deberíamos sospe- tanto el hemotórax como el neumopericardio cursan con tonos cardiacos apa-
char que se trata de una enfermedad de membrana hialina, lo que además gados (hay algo entre el corazón y el fonendo, bien aire o bien sangre, que difi-
coincide con la clínica y la radiografía que nos presentan. Una vez que el tra- cultan que llegue claramente el sonido). También es verdad que en la enfer-
tamiento de mantenimiento falla se debe pasar a aportar lo que les falta, es medad pulmonar crónica o fibrosis pulmonar intersticial, la principal causa de
decir, el líquido surfactante administrado intratraquealmente. muerte es la insuficiencia cardiaca y la bronquiolitis obliterante. La respuesta
incorrecta se refiere a la displasia broncopulmonar que es una enfermedad pul-
27. Señale cuál de las siguientes anormalidades se produce en los RNPT con dis- monar que aparece cuando los niños han estado expuestos a ventilación mecá-
nica y fundamentalmente al efecto tóxico del oxígeno.
trés respiratorio idiopático o síndrome de las membranas hialinas, como conse-
cuencia de las alteraciones fisiopatológicas típicas de este síndrome:
30. ¿Cuál es la cardiopatía congénita más frecuente?
1. Hipoplasia del corazón izquierdo
2. Apertura de shunts arteriovenosos cerebrales 1. Comunicación interventricular (CIV)
3. Hipoplasia de ventrículo derecho 2. Comunicación interauricular (CIA)
4. Hipertensión arterial sistémica 3. Transposición de los grandes vasos (TGV)
5. Persistencia del ductus arterioso y del foramen oval 4. Tetralogía de Fallot
5. Drenaje venoso pulmonar anómalo (DVPA)
Esta pregunta también corresponde a un examen MIR y siguiendo con la enfer-
medad de membrana hialina nos preguntan por una alteración relativamente Esta pregunta se refiere a las cardiopatías congénitas, de las cuales debes cono-
frecuente que es consecuencia de las modificaciones fisiopatológicas típicas de cer algunos datos sobre su frecuencia:
este síndrome: la persistencia del ductus arterioso y del foramen oval. Debemos La más frecuente de todas → válvula aórtica bicúspide, aunque generalmen-
recordar que, mientras el niño sufre la enfermedad, los pulmones son una cir- te no figura entre las opciones de respuesta y sin embargo sí está la siguien-
culación de alta resistencia que favorecen que no pase la sangre a través de te en frecuencia, la CIV.
ellos, y que ésta se desvíe por el conducto arterioso hacia la aorta y por el fora- La más frecuente de las cianóticas → tetralogía de Fallot, aunque realmente
men oval de la aurícula derecha a la izquierda. Cuando el niño mejora de su no se suele convertir en cianótica hasta el año de vida.
enfermedad, los pulmones disminuyen su resistencia al flujo de sangre pero Cianótica más frecuente al nacimiento → transposición de grandes vasos.
como el conducto arterioso y el foramen oval permanecen abiertos pasa san-
gre en sentido inverso y los pulmones sufren un hiperaflujo sanguíneo. 31. En relación a las cardiopatías congénitas con cortocircuito izquierda-dere-
cha (arterio-venoso), señale la afirmación que considera INCORRECTA:
28. Un neonato de 36 semanas de edad gestacional y 24 horas de vida presen-
ta pocas horas después de su nacimiento taquipnea, aleteo nasal, retracciones 1. Constituyen el 40% de todas las cardiopatías
inter y subcostales y ligera cianosis. En la radiografía de tórax aparece un pul- 2. Es posible un enfoque diagnóstico con la clínica y la radiología sim-
món hiperinsuflado, con diafragmas aplanados, refuerzo hiliar y líquido en cisu- ple
ras. ¿Cuál sería la actitud adecuada? 3. La hipertensión pulmonar es una complicación potencialmente
grave
1. Administrar una dosis de surfactante endotraqueal e iniciar anti- 4. La crisis hipoxémica es la complicación aguda más frecuente
bioterapia empírica con ampicilina y gentamicina 5. La cirugía ofrece buenos resultados evolutivos
2. Ventilación con PPI
3. Oxigenoterapia, analítica, cultivos y observación De nuevo nos encontramos con una pregunta de examen MIR, ésta se refiere
4. Estabilización hemodinámica y estudio cardiológico a cardiopatías congénitas y, aunque no sepamos nada de ninguna de las car-
5. Oxigenoterapia, reposición hidroelectrolítica y antibioterapia diopatías, con unas pocas ideas sobre su fisiopatología es suficiente para res-
ponder. Debes tener claro que existen, a grandes rasgos, dos tipos de cardio-
patías: las cianógenas (cortocircuito derecha – izquierda) y las no cianógenas
Esta pregunta pertenece también a un examen MIR y continúa con los cuadros
(cortocircuito izquierda – derecha). En relación a las cardiopatías congénitas
respiratorios. En este caso el factor de riesgo no está claro, probablemente
con cortocircuito izquierda-derecha (arterio - venoso), debes recordar que el
según la radiografía estamos ante un pulmón húmedo o taquipnea transitoria
hecho de que pase sangre del lado izquierdo del corazón al derecho supone
del recién nacido. Está claro que si estamos seguros del diagnóstico no sería
una situación de hiperaflujo pulmonar, lo que condiciona que se hipertrofien
necesario sacar cultivos, aunque en general siempre que se tengan dudas razo-
los vasos pulmonares y se produzca el denominado síndrome de
nables se debe descartar que el recién nacido esté infectado ya que podrían ser
Einsenmenger, en el que el grado de hipertensión pulmonar va en aumento
muy graves las consecuencias si no se descarta y trata una posible infección. En
hasta que se produce una situación de insuficiencia cardiaca derecha que obli-
este caso no es necesario administrar surfactante endotraqueal porque con esa
ga a realizar un trasplante cardiopulmonar. Con respecto a las cardiopatías cia-
edad de gestación el niño no carece de él. Si el distrés no es muy importante
nógenas es importante conocer que la sangre pasa del lado derecho al izquier-
es suficiente con oxigenoterapia o como mucho con presión continua en la vía
do, lo que provoca una situación de hipoaflujo pulmonar e hipoxia crónica,
aérea, en principio no sería necesaria PPI (presión positiva intermitente). En la
esto hace que los tejidos detecten la falta de oxígeno y se produzca una poli-
pregunta no nos cuentan ningún dato de inestabilidad hemodinámica y por lo
citemia reactiva. Con esto queda claro que la respuesta incorrecta en este caso
tanto no es necesario estabilizarlo ni realizar estudio cardiológico. La última de
es la que dice que la crisis hipoxémica es la complicación aguda más frecuente
las respuestas podría considerarse válida pero quizá es más correcto realizar
de las cardiopatías con cortocircuito izquierdo – derecho. El resto de las opcio-
analítica y sacar los cultivos antes de poner antibióticos. nes son correctas, no es necesario conocer porcentajes pero es verdad que son
más frecuentes las cardiopatías cianógenas. Sabiendo si el niño se pone o no
29. Con respecto a la patología respiratoria del recién nacido, señale la res- cianótico (clínica) y viendo si hay hiperaflujo pulmonar o no (radiografía de
puesta INCORRECTA: tórax) se podría orientar el diagnóstico. En general, la cirugía resuelve bastan-
te bien los cuadros.
1. La hipertensión pulmonar persistente se produce cuando no hay

5 ] PEDIATRÍA [
www.academiamir.com Comentarios test de clase A Mir

32. Ingresa en su unidad de cuidados intensivos neonatales un recién nacido Esta pregunta entra dentro de la patología digestiva del recién nacido. Cuando
cianótico que no responde a la aplicación de O2 100%, usted le ausculta pero un niño de esta edad vomita, una de las causas más importantes que hay que
no aprecia ningún soplo. El diagnóstico más probable es: descartar es una obstrucción como consecuencia de una atresia intestinal. Lo
más frecuente es que dicha obstrucción esté localizada a nivel del esófago o
del intestino delgado. En general suele haber antecedente prenatal de polihi-
1. Conducto arterioso persistente dramnios ya que el feto participa de la fabricación de su propio líquido amnió-
2. Síndrome de aspiración meconial tico, lo deglute y lo orina, por tanto si hay un problema digestivo el niño no lo
3. Transposición de las grandes arterias traga y se acumula (polihidramnios). El recién nacido no tiene salivación activa
4. Hemorragia intracraneal (no babea, como el lactante) y por tanto si hay mucha saliva hay que sospechar
5. Coartación aórtica que haya una obstrucción, fundamentalmente en el esófago. A las 24 horas de
vida debe haber gas distal visible en la radiografía y si este no está presente se
debe sospechar obstrucción intestinal. La respuesta incorrecta es la última por-
Esta pregunta también corresponde a una de un examen MIR. Como nos
que la imagen de doble burbuja es típica de obstrucción a nivel del duodeno
enfrentamos a ella en el tema de las cardiopatías congénitas nos vamos direc-
(lo típico de obstrucciones en el píloro es la burbuja única) y además la EHP no
tos a contestar que la respuesta correcta es la transposición de las grandes arte-
rias, pero claro, si nos encontramos esta pregunta en el contexto de un exa- se manifiesta desde recién nacido, sino desde los 21 días de vida.
men completo hacer el diagnóstico diferencial entre las 5 respuestas sería un
poco más complicado. El enunciado nos dice que se trata de algún tipo de pro- 36. ¿Cuál de las siguientes NO le parece una causa de estreñimiento?
blema cianógeno, esto ya nos descarta las opciones 1 y 5 que no lo son. Con
respecto a la opción 2, el niño debería tener clínica de dificultad respiratoria, 1. Atresia intestinal
ya que es esta la causa de su cianosis. La respuesta 4 queda descartada porque 2. Acalasia neonatal
no me cuentan nada en el enunciado que me lleve a pensar en que el niño ha
3. Enfermedad de Hirschprung
sangrado (parto traumático, situación de hipoxia, etc).
4. Hipotiroidismo
5. Estenosis anal
33. ¿Cuál es el límite para indicar cirugía en un caos de síndrome de Down con
canal atrioventricular completo y aumento de la presión pulmonar? Ésta es una pregunta directa sobre las causas de estreñimiento, todas las opcio-
nes salvo la acalasia, que es un problema de la deglución (el alimento degluti-
1. Final del periodo neonatal do no progresa), son causas de estreñimiento.
2. Antes de los 12 meses
3. Antes de los 18 meses 37. ¿Cuál de las siguientes NO le parece que sea una causa de retraso en la eli-
4. Antes de los 24 meses minación del meconio?
5. Antes de los 36 meses
1. Preeclampsia
Esta pregunta, con bastante mala idea por cierto, también es una pregunta de 2. Síndrome de colon izquierdo hipoplásico
examen MIR. Se refiere a una cardiopatía bastante compleja que tienen en
3. Fibrosis quística
algunas ocasiones los niños con síndrome de Down y en la que se observan una
CIA tipo ostium primun, un canal atrioventricular, una CIV y una válvula 4. Agangliosis rectal
común. Dada la complejidad de la situación, se debe intervenir relativamente 5. Drogadicción materna
pronto pero no en le periodo neonatal para permitir cierre de conductos y darle
un poco de tiempo a que el corazón crezca y sea posible realizar anastomosis Ésta también es una pregunta directa en la que nos piden que localicemos
vasculares y suturas intracardiacas con el menor riesgo posible para el pacien- entre las respuestas la que no es una causa de retraso en la eliminación del
te (antes del año de vida). meconio. Todas menos la preeclampsia lo son, aunque habría que recordar que
el fármaco utilizado para tratar la preeclampsia (sulfato de magnesio) también
34. ¿Qué cardiopatía presenta un niño de 6 meses si tiene un soplo eyectivo en es una causa de retraso en la eliminación del meconio.
borde esternal, volumen cardiaco normal, vascularidad pulmonar disminuida,
hipertrofia derecha en el ECG y los siguientes datos de laboratorio: 95000 leu- 38. Un lactante de 3 meses de vida presenta desde hace un mes episodios inter-
cocitos/mcl con fórmula normal, hematíes 7 mill/mcl, Hb 16g/dl y Hct 70? mitentes de distensión abdominal, dolores de tipo cólico, algunos vómitos y
tendencia al estreñimiento. Entre sus antecedentes personales destaca que fue
1. Estenosis valvular aórtica prematuro y que tuvo dificultad respiratoria que precisó ventilación mecánica
2. Estenosis pulmonar valvular durante 15 días. ¿Cuál es su diagnóstico según este cuadro clínico?
3. Tetralogía de Fallot
4. Atresia tricuspídea 1. Megacolon congénito
5. Atresia pulmonar 2. Vólvulo intestinal intermitente
3. Enteritis crónica por rotavirus
Esta pregunta también se refiere a una cardiopatía congénita y también perte- 4. Adenitis mesentérica intermitente
nece a un examen MIR. En primer lugar debes recordar que los soplos eyecti- 5. Estenosis cólica secundaria a enterocolitis necrotizante
vos, piantes, rudos, fuertes y paraesternales suelen referirse a CIV. En segundo
lugar hay datos de hipoaflujo pulmonar (podría corresponder a una estenios Ésta es una pregunta de examen MIR y sirve para recordar el cuadro gastroin-
pulmonar o a un cortocircuito derecha izquierda), de hipoxia crónica (policite- testinal más grave que puede padecer un prematuro: la enterocolitis necroti-
mia) y de hipertrofia de ventrículo derecho, todas características comunes a la zante. En este caso no nos cuentan el caso típico de una enterocolitis necroti-
tetralogía de Fallot. Además debes tener en cuenta que el niño presenta una zante sino las consecuencias de haber padecido la enfermedad. Recuerda que
cardiopatía de las de hipoaflujo pulmonar y aun (6 meses) no se ha puesto cia- en el MIR no dan datos al azar y en este caso nos cuentan que el niño fue pre-
nótico, esto es típico de un Fallot. También podrías descartar algunas de las maturo y tuvo dificultad respiratoria que precisó ventilación mecánica durante
opciones porque estas no hipertrofian el ventrículo derecho, así serían inco- 15 días, lo cual provocó situaciones de isquemia en todo su organismo, espe-
rrectas la 1 y la 4. Por último, y aunque no sea muy ortodoxo, la pregunta se cialmente en su intestino. Las áreas de mucosa intestinal que quedaron daña-
responde teniendo en cuenta que de todo lo que hay en las respuestas, la única das y no fueron resecadas han dado lugar a zonas estenóticas que son las que
cardiopatía que os hemos dicho que debéis conocer es la tetralogía de Fallot. provocan los episodios de obstrucción durante varios meses después. El mega-
colon congénito y el vólvulo intestinal intermitente podrían provocar obstruc-
35. Un recién nacido presenta a las pocas horas del nacimiento vómitos abun- ción pero podrían aparecer en cualquier niño y no serían típicos de prematu-
dantes pero no proyectivos, distensión abdominal y ondas peristálticas visibles. ros. No existe la enteritis crónica por rotavirus, lo que se suelen producir son
episodios de diarreas agudas autolimitadas. La adenitis mesentérica es un cua-
Sobre la patología que sospecha ¿cuál de las siguientes afirmaciones es INCO- dro que se confunde con la apendicitis aguda y es típico de niños mayores, no
RRECTA? de lactantes.

1. Lo más frecuente es que esté localizada a nivel del esófago o del 39. ¿Cuál de las siguientes enfermedades se caracteriza fundamentalmente por
intestino delgado una hiperbilirrubinemia mixta?
2. Puede haber antecedente prenatal de polihidramnios
3. Si la localización es en el esófago, el recién nacido puede tener sali- 1. Enfermedad de Alagille
vación constante 2. Enfermedad de Caroli
4. En la radiografía se puede observar ausencia de gas distal a las 24 3. Enfermedad de Dubin-Jonhson
horas de vida 4. Déficit de α1-antitripsina
5. Si en la radiografía se aprecia una imagen de doble burbuja, lo más 5. Galactosemia.
probable es que se trate de una estenosis pilórica

6 ] PEDIATRÍA [
www.academiamir.com Comentarios test de clase A Mir

Esta es una pregunta directa, sólo para que recuerdes que la enfermedad que recordar que en el recién nacido, y especialmente en el prematuro, puede exis-
se caracteriza fundamentalmente por una hiperbilirrubinemia mixta es la enfer- tir hipocalcemia. Entre las respuestas que aparecen sólo la dieta rica en fosfatos es
medad de Dubin-Jonhson. Del resto recuerda que son causas de hiperbilirrubi- causa de hipocalcemia tardía, el resto son causas de hipocalcemia precoz.
nemia directa y cuál es la fisiopatología (aunque se verá de forma más detalla-
da en digestivo). La enfermedad de Alagille es una hipoplasia de los conductos 44. Acude usted a paritorio porque acaba de nacer un niño de 38 semanas de
intrahepáticos, la enfermedad de Caroli es una dilatación de los conductos
intrahepáticos, el déficit de α1-antitripsina provoca éstasis hepática y la galac-
edad de gestación que pesa 4.200g. Es el primer hijo de una mujer aparente-
tosemia citolisis.
mente sana aunque el embarazo no ha sido controlado. Usted sospecha que el
recién nacido puede tener algunas complicaciones en las próximas horas ¿cuál
40. ¿Cuál de las siguientes no es una característica de la ictericia fisiológica? NO esperaría encontrar?

1. Bilirrubina directa >3 mg/dl 1. Asimetría del tabique cardiaco


2. Bilirrubina indirecta en sangre de cordón = 3 mg/dl 2. Hipoglucemia
3. Inicio a partir de las primeras 24 horas de vida 3. Agenesia lumbosacra
4. Duración inferior a 7 días 4. Anemia
5. Ausencia de patología de base 5. Trombosis de la vena renal

Esta pregunta hace referencia a otra de las posibles complicaciones metabóli-


Esta pregunta sirve para recordar una de las causas de ictericia benigna, la icte-
cas que puede padecer un recién nacido. Cuando nazca un niño macrosómico,
ricia fisiológica. Para responderla recuerda las cuáles son las características que
una de las cosas que hay que sospechar es que sea hijo de madre diabética, y
debe cumplir una ictericia para pertenecer a este cuadro:
más si nos dicen que la madre no se controló el embarazo. Con esta sospecha
(2) Bilirrubina directa ≤ 2mg/dl
clínica, todas las respuestas que nos dan salvo la anemia son complicaciones
(3) Bilirrubina indirecta en sangre de cordón ≤ 3mg/dl
(5) Incremento de bilirrubina total ≤ 5mg/dl/día (≤ 0,5mg/dl/h) que pueden padecer estos niños.
(7) Duración inferior a 7 días (RNPT más prolongado)
(12) Bilirrubina total ≤ 12mg/dl en RNT (≤ 14mg/dl en RNPT) 45. Le avisan de una planta de la maternidad porque tienen una niña de 3 horas
(24) Inicio después de las 24 primeras horas de vida (RNPT más tardío) de vida con una facies "peculiar". En la historia observa que ha nacido a las 39
Ausencia de patología de base semanas de EG y que ha pesado 4.500g. La facies es tosca, los párpados y
labios están tumefactos, la nariz es corta y la base está deprimida, tiene macro-
41. Con respecto a la incompatibilidad materno-fetal en el sistema ABO de gru- glosia e hipertelorismo y en el abdomen se aprecia una hernia umbilical. Con
pos sanguíneos, es cierto que: respecto a la patología que sospecha, señale cuál de las siguientes afirmacio-
nes le parece INCORRECTA:
1. Es más grave que la isoinmunización por incompatibilidad de Rh.
2. Afecta a primogénitos 1. La incidencia ha disminuido gracias a la realización de screening
3. La detección prenatal es muy importante porque se asocia a ane- con pruebas de detección precoz
mia fetal severa 2. Las pruebas deben realizarse entre las 48 horas y el 5º día de vida,
4. La prueba de Coombs es positiva antes de que empiece la alimentación proteica
5. Es una causa muy infrecuente de enfermedad hemolítica del recién 3. Es característica la dificultad para la alimentación, con episodios de
nacido sofocción durante la lactancia
4. El retraso en el desarrollo físico y mental se acentúa en los prime-
Esta pregunta pertenece a un examen MIR y en realidad dos de las respuestas ros meses
(la 2 y la 4) podrían considerarse correctas. La incompatibilidad materno-fetal 5. La dentición también está retrasada
en el sistema ABO de grupos sanguíneos es mucho más leve y frecuente que la
isoinmunización por incompatibilidad de Rh y generalmente no se asocia a ane-
En esta pregunta nos reflejan fielmente las características fenotípicas que pre-
mia fetal severa. No es necesario que la madre se inmunice, ya posee anticuer-
sentaría una recién nacida con hipotiroidismo congénito. Con respecto a la
pos frente al grupo sanguíneo del feto y por tanto puede afectar a primogéni-
enfermedad hay que recordar que la incidencia ha disminuido gracias a la rea-
tos. La prueba de Coombs, especialmente la indirecta aunque también en algu-
lización de screening con pruebas de detección precoz, que es cierto que se
nos casos la directa, es positiva.
realizan entre las 48 horas y el 5º día de vida, pero que no tienen valor si se
realizan antes de que empiece la alimentación proteica. El resto de las res-
42. Con respecto a la enfermedad hemorrágica del recién nacido, en la forma puestas son características típicas de la enfermedad.
clásica de déficit de vitamina K, NO es cierto que:
46. ¿Cuál de las siguientes características no es típica de una sepsis precoz?
1. Aparece en las primeras 24-48 horas de vida
2. Se debe a un déficit de factores vitamina K dependientes (II, VII, IX, 1. El agente causal más frecuente es el Streptococo agalactiae
XI) 2. Aparece en los primeros 7 días de vida
3. Su incidencia ha disminuido gracias a la administración de vitami- 3. Suele cursar como una afectación multisistémica fulminante
na K de forma profiláctica al recién nacido 4. Causa meningitis en el 95% de los casos
4. Es mucho más frecuente en los niños alimentados con fórmula 5. El tratamiento suele ser ampicilina y gentamicina
artificial
5. En algunas ocasiones la clínica se prolonga más allá del tercer día
SEPSIS PRECOZ SEPSIS TARDÍA
De las posibles hemorragias que puede padecer un recién nacido, ésta es la
causa más importante y esta pregunta es para recordar sus características.
< 7 dias de vida > 7 dias de vida
Debes saber que la enfermedad hemorrágica del recién nacido, en la forma clá-
sica de déficit de vitamina K, aparece en las primeras 24-48 horas de vida y en Ascendente; Postnatal; invasión por gérmenes
algunas ocasiones la clínica se prolonga más allá del tercer día. Se debe a un gérmenes del canal de parto colonizadores de via respiratoria y piel
déficit de factores vitamina K dependientes (II, VII, IX, XI) y es verdad que su SGB serotipo III, E. coli K1 (meningi-
incidencia ha disminuido gracias a la administración de vitamina K de forma SGB, E. coli, enterococ, Listeria, VHS, tis), Listeria, VHS, S. epidermidis y
profiláctica al recién nacido. Es falso que sea mucho más frecuente en los niños Enterovirus Candida en ENMBP
alimentados con fórmula artificial ya que es típica de niños alimentados con
lactancia materna (deficitaria en vitamina K). RNPT, bajo peso,
RNPT, hospitalización
RPMSBG+ materno, corioamnionitis
43. Señale cual de las siguientes no es una causa de hipocalcemia precoz en el Afectación multisistémica fulminante Meningitis, ostiomielitis, artritis, ITUs
recién nacido:
Meningitis 20% Meningitis 75%
1. Diabetes materna
2. Pretérmino Ampicilina + Gentamicina
Ampicilina + Gentamicina
Vancomicina + Gentamicina
3. Hipoxia Ampicilina + Cefotaxima
(si estaba ingresado)
4. Dieta rica en fosfatos (si meningitis)
Ampicilina + Cefotaxima
5. Síndrome de Di George 7-10 dias; 14 para SGB, 21 para BGN
(si meningitis)
Alta morbilidad (hidrocefalia, sordera,
Ésta es una pregunta directa y muy poco importante, fundamentalmente debes Alta mortalidad
retraso mental, PCI)

7 ] PEDIATRÍA [
www.academiamir.com Comentarios test de clase A Mir

En esta pregunta lo fundamental es que tengas claras las diferencias entre una Esta pregunta también es directa y es para recordar que lo típico del toxoplas-
sepsis precoz y una tardía en el recién nacido. La precoz, por definición es ma Gondii congénito es la tetrada de Sabin (coriorretinitis, hidrocefalia, calcifi-
aquella que aparece en los primeros 7 días de vida. Debes recordar que en caciones y convulsiones).
ambos casos el agente causal más frecuente es el Streptococo agalactiae.
Cuando la infección aparece de forma precoz, suele cursar como una afecta- 51. La triada de Hutchinson (sordera laberíntica, queratitis y alteraciones den-
ción multisistémica fulminante y es raro que produzca una infección localizada
como la meningitis (típico de una sepsis tardía). El tratamiento de elección son
tarias) es típica de:
los antibióticos de amplio espectro intravenosos (ampicilina y gentamicina).
1. CMV congénito
47. Con respecto a la neumonía del recién nacido: 2. Parvovirus congénito
3. Toxoplasma Gondii congénito
1. La clínica suele ser inespecífica, con letargia, rechazo de tomas, irri- 4. Sífilis congénita
tabilidad, mal color, distensión abdominal, etc 5. Rubéola congénita
2. La adquirida en la comunidad cursa con clínica de mayor gravedad.
De nuevo una pregunta directa sobre las TORCH, en este caso hace referencia
3. Los hemocultivos son útiles para determinar el germen y su sensi-
a la triada de Hutchinson (sordera laberíntica, queratitis y alteraciones denta-
bilidad rias) que es típica de sífilis congénita.
4. El lavado bronquioalveolar es una prueba bastante fiable
5. Existen serologías para la mayor parte de los gérmenes que afec- 52. Con respecto a la infección congénita por VHS, señale la afirmación que le
tan al recién nacido parezca correcta:
En esta pregunta se hace referencia a otra infección del recién nacido, muy fre-
1. El parto debe ser por cesárea, dado el alto riesgo de contagio
cuente pero con menor relevancia, en lo que a preguntas se refiere, para el
MIR. En general debes recordar que cualquier infección importante y sistémica 2. No es necesario aislar al recién nacido del resto ya que para el con-
en el recién nacido se manifiesta de manera inespecífica, con letargia, rechazo tagio se requiere un contacto muy directo
de tomas, irritabilidad, mal color, distensión abdominal, etc. Los agentes cau- 3. La lactancia materna está contraindicada en todos los casos
sales de la neumonía adquirida en la comunidad son menos virulentos y más 4. El diagnóstico se hace mediante raspado de vesículas y cultivo (téc-
fáciles de tratar y normalmente cursa con clínica de menor gravedad que la nica muy específica)
nosocomial. El diagnóstico suele basarse en la clínica, la analítica y las radio- 5. El tratamiento más adecuado es el aciclovir vía oral
grafías y ni los hemocultivos, ni el lavado bronquioalveolar ni las serologías son
demasiado útiles.
Esta es una pregunta en la que figuran algunas de las características típicas de
la infección congénita por VHS. El parto debe ser vaginal salvo que existan
48. Nace en su hospital un niño que presenta microcefalia. A las pocas horas lesiones activas a nivel genital, en ese caso se debe realizar una cesárea. Dado
del nacimiento presenta petequias y se le palpa una hepatoesplenomegalia que se trata de una infección grave y altamente contagiosa, es necesario aislar
moderada. Se realiza una ecografía transfontanelar en la que se observan cal- al recién nacido del resto de los niños ingresados en la unidad de neonatolo-
cificaciones periventriculares. Con respecto a la enfermedad que usted sospe- gía. La lactancia materna está contraindicada sólo en los casos en los que exis-
cha, señale la opción INCORRECTA: tan lesiones activas a nivel de la areola mamaria. El diagnóstico se hace median-
te raspado de vesículas y cultivo (técnica muy específica) y el tratamiento más
adecuado es el aciclovir vía intravenosa.
1. Es la infección connatal más frecuente
2. La infección intraútero es la más frecuente, aunque también puede
53. Le avisan a paritorio porque ha nacido el hijo de una madre toxicómana en
darse postnatalmente mediante lactancia, contacto interpersonal,
tratamiento de desintoxicación con metadona (40 mg/día). Señale cual de las
hemoderivados o sangre
siguientes afirmaciones es INCORRECTA:
3. Si la infección se produce de forma vertical, cursa con afectación
multisistémica, con la clínica típica del síndrome de TORCH
1. La dosis recibida por la madre es muy baja, lo normal es que no
4. La prueba más sensible para el diagnóstico es la detección de CMV
aparezcan síntomas en el recién nacido
en sangre
2. El síndrome de abstinencia es tardío, aparecerá hacia las 2-4 semanas
5. Si la IgG es negativa en la madre y en el recién nacido se excluye
3. Puede haber algunos síntomas al nacimiento que mejoran rápida-
la infección congénita
mente
4. La clínica fundamental consiste en alteraciones del sueño, con-
Esta es la primera de una serie de enfermedades que están incluidas entre las
de tipo TORCH o infecciones congénitas. Debes saber que detalle en concreto ducta hiperactiva, temblor, sudoración, hipertonía, convulsiones, etc.
te permite identificar el agente causal. El truco en esta pregunta son las calci- 5. El tratamiento es fundamentalmente sintomático, aunque en fun-
ficaciones periventriculares, típicas de CMV congénito. Es cierto que es la infec- ción de la clínica se administrará morfina asociada o no con fenobar-
ción connatal más frecuente y que generalmente se produce por infección bital
intraútero, aunque también puede darse postnatalmente mediante lactancia,
contacto interpersonal, hemoderivados o sangre. Para el diagnóstico, la prue- Esta pregunta hace referencia a las toxicomanías en mujeres gestantes y como
ba más sensible es la detección del antígeno de CMV en orina (NO en sangre). afectan al recién nacido. Debes conocer algunos datos sobre la metadona, la
Si la IgG es negativa en la madre y en el recién nacido, se excluye la infección heroína y la cocaína. En general no es necesario que conozcas dosis, en este
congénita. caso la cantidad de metadona recibida por la madre es el doble de la conside-
rada causante de mínimos síntomas (20 mg/día). El síndrome de abstinencia en
49. La denominada triada de Gregg (catarata, sordera y cardiopatía congénita) el caso de la metadona es más tardío que el de la heroína, aparecerá hacia las
es típica de: 2-4 semanas, aunque puede haber algunos síntomas al nacimiento que mejo-
ran rápidamente. La clínica fundamental es la de un niño que está pasando un
mono (alteraciones del sueño, conducta hiperactiva, temblor, sudoración,
1. CMV congénito
hipertonía, convulsiones, etc). El tratamiento es fundamentalmente sintomáti-
2. Parvovirus congénito co, aunque si la clínica es muy intensa se administrará morfina asociada o no
3. Toxoplasma Gondii congénito con fenobarbital.
4. Sífilis congénita
5. Rubéola congénita 54. Con respecto al crecimiento y desarrollo normal de un niño, señale la afir-
mación INCORRECTA:
Esta es una pregunta directa que se refiere a la rubéola congénita, recuerda
que suele cursar con la denominada triada de Gregg y cuáles son las caracte-
1. En las últimas décadas se ha observado un aumento de la talla
rísticas que la forman (catarata, sordera y cardiopatía congénita).
media poblacional, secundario a un mejor equilibrio en el aporte
calórico-proteico
50. La tetrada de Sabin (coriorretinitis, hidrocefalia, calcificaciones y convulsio-
2. El índice más sensible para detectar alteraciones en el crecimiento
nes) es típica de:
es el índice de masa corporal
3. Como regla general, un niño duplica su peso de recién nacido a
1. CMV congénito
los 6 meses de vida y lo triplica al año
2. Parvovirus congénito
4. Para valorar la edad ósea se utiliza la radiografía de tibia izquierda
3. Toxoplasma Gondii congénito
en menores de 1 año y la de muñeca izquierda en mayores de 1 año
4. Sífilis congénita
5. La dentición comienza por los incisivos centrales hacia los 6-8
5. Rubéola congénita
meses y avanza hacia los lados

8 ] PEDIATRÍA [
www.academiamir.com Comentarios test de clase A Mir

Esta pregunta sirve para recordar algunas generalidades sobre el crecimiento y dad es que tiene bastante mala idea por lo específica que es. En el manual
desarrollo normal de un niño, la mayor parte de las respuestas son bastante (página 39) figura el siguiente cuadro con las principales diferencias entre
fáciles de deducir con un poco de sentido común. Es obvio que en las últimas ambas leches, que es lo que tienes que saber sobre este tema.
décadas se ha observado un aumento de la talla media poblacional y esto es
debido a un mejor equilibrio en el aporte calórico-proteico. Como regla gene-
ral debes recordar que un niño duplica su peso de recién nacido a los 6 meses LM LA
de vida y lo triplica al año. El índice más sensible para detectar alteraciones en
el crecimiento es la velocidad de crecimiento, el índice de masa corporal es el ↑

índice más útil para definir obesidad. El resto de las opciones son ciertas: para ß-lactoalbúmina
PROTEÍNAS α-lactoalbúmina
valorar la edad ósea de un niño se utiliza la radiografía de tibia izquierda en 80% caseína
75% caseína
menores de 1 año y la de muñeca izquierda en mayores de 1 año. Recuerda ++ Fenilalanina
que la dentición comienza por los incisivos centrales hacia los 6-8 meses y avan-
za hacia los lados (igual que cuando se nos caen hacia los 6-8 años, que tam- ↑
bién empiezan por los incisivos). LACTOSA (10% polisacáridos ↓
y glucoproteínas)
55. Señale cuál de los siguientes hitos del desarrollo NO aparece en el momen- = =
to que se indica a continuación: GRASAS AG esenciales AG saturados,
e insaturados se digieren peor

1. 1 mes: sonrisa social CALORIAS 0.67 Kcal/ml 0.67 Kcal/ml


2. 3 meses: sostén cefálico
+++
3. 6 meses: sedestación MINERALES +
(excepto hierro y cobre)
4. 7 meses: reptación
5. 12 meses: deambulación HIERRO + (se absorbe mejor) +

Al responder esta pregunta observa que el desarrollo normal de un niño COBRE ++ +


comienza por la cabeza y avanza hacia los pies. Debes recordar fundamental- FLÚOR + -
mente los siguientes hitos:
RN: fija la mirada CA/P 2 (40/20) 1 (125/96)
1 mes: sonrisa social
3 meses: sostén cefálico VITAMINAS C, D, A, B, E K, A, B, E
6º mes: sedestación
DIGESTIBILIDAD Mayor vaciado gástrico Menor vaciado gástrico
9º mes: comienza la reptación
12º mes: deambulación, primeras palabras
58. La aparición de una metahemoglobinemia en el lactante pequeño puede
56. Señale cuál de las siguientes características NO corresponde a un retraso estar en relación con la introducción precoz en la alimentación de:
constitucional del crecimiento:
1. Pollo
1. Peso y talla por debajo del p3 hasta el final de la lactancia 2. Espinacas
2. Aceleración del crecimiento al final de la adolescencia 3. Gluten
3. Talla adulta generalmente normal 4. Yema de huevo
4. Edad ósea igual que edad cronológica 5. Pescado
5. Historia familiar de retraso constitucional
Esta también es una pregunta de examen MIR que se refiere a una complica-
En esta pregunta se hace referencia a dos cuadros que cursan con talla baja y ción muy concreta sobre la alimentación complementaria. Recuerda que la apa-
que se consideran variantes de la normalidad: el retraso constitucional del cre- rición de una metahemoglobinemia en el lactante pequeño puede estar en
cimiento y la talla baja familiar: relación con la introducción precoz en la alimentación de espinacas.

RETRASO CONSTITUCIONAL 59. Señale cuál de las siguientes características es típica de la malnutrición caló-
TALLA BAJA FAMILIAR
DEL CRECIMIENTO rica o marasmo:
Más frecuente
1. Déficit fundamentalmente proteico
2. El peso y la talla se disminuyen precozmente
Peso y talla <p3 Peso y talla <p3 3. El niño se manifiesta letárgico, apático
4. Pueden tener diarrea de la emanciación, aunque lo más frecuente
Curva paralela en lactancia,
Curva paralela es que tengan estreñimiento
aceleración adolescencia
5. El pelo suele ser escaso, fino y de color rojizo
Talla adulta normal Talla final baja
Sobre la malnutrición hace muchos años que no caen preguntas en el MIR, si
te sobra tiempo de estudio echa un vistazo a las principales diferencias entre la
EO < EC EO = EC malnutrición calórica o marasmo y la proteica o kwashiorkor (manual, página
40). Todas las respuestas pertenecen al kwashiorkor salvo la 4 que es clásica-
Hia familiar Hia familiar mente del marasmo.

Laboratorio: N Laboratorio: N 60. Acude a su consulta una niña de 9 años que consulta por aumento de peso
desde los 6 años, aunque acompañado de un aumento de talla importante. Los
padres, que son obesos, refieren buen apetito con ingesta consecuente de ali-
Por tanto, la edad ósea en el retraso constitucional del crecimiento debe ser mento y tendencia a la inactividad. En la exploración física se observa un índi-
menor que la edad cronológica. ce de masa corporal de 158%, telarquia bilateral 2/4, pubarquia 3/6 e inicio de
vello axilar. Señale el posible diagnóstico y las pruebas complementarias que
57. ¿Cuál de las siguientes afirmaciones es cierta con respecto a la leche mater- solicitaría:
na?
1. Pubertad precoz idiopática. FSH y LH para distinguir si es central o
1. Contiene más sales que la leche de vaca periférica
2. Contiene menos fenilalanina que la leche de vaca 2. Pubertad precoz secundaria a la obesidad. FSH, LH y 17 beta-estradiol,
3. Contiene más ácidos grasos de cadena corta que la leche de vaca y si los valores son elevados iniciaría tratamiento con andrógenos
4. Contiene más calcio que la leche de vaca 3. Obesidad y adelanto puberal secundario. FSH, LH, 17 beta-estra-
5. Contiene menos nitrógeno proteico diol y maduración esquelética para comprobarlo. Iniciaría restricción
calórica y reevaluaría a los 6 meses
Esta es una pregunta directa sobre las diferencias entre la lactancia artificial 4. Adelanto puberal y obesidad como problemas independientes.
(similar a la leche de vaca) y la materna. Pertenece a un examen MIR y la ver- Tras un estudio de la función tiroidea y de FSH y LH, iniciaría trata-

9 ] PEDIATRÍA [
www.academiamir.com Comentarios test de clase A Mir

miento con progestágenos para frenar la pubertad que tiene exceso de salivación, debes recordar que el recién nacido, a diferen-
5. Obesidad y adelanto puberal, ambas por un posible hipotiroidis- cia del lactante, no tiene salivación activa y si ésta aparece hay que sospechar
mo. Solicitaría T4L y TSH y, aunque los valores sean normales, si la trastornos de tipo obstructivo alto (atresias esofágicas). El diagnóstico se haría
intentando pasar una sonda nasogástrica hasta el estómago y, mientras se rea-
maduración esquelética está atrasada iniciaría tratamiento sustitutivo
liza la cirugía, el niño debe estar en decúbito supino. Si el niño no fuese un
recién nacido, la salivación continua haría pensar en infecciones del territorio
Esta pregunta pertenece a un examen MIR pero no es del todo correcto su ORL, faringoamigdalitis complicadas (abscesos) o epiglotitis.
enunciado. Nos cuentan que una niña de 9 años consulta por aumento de peso
durante los últimos 3 años. En la historia existen datos para sospechar que se
trata de una obesidad de causa exógena, es decir, seguramente la niña no 65. En relación con la fístula traqueoesofágica congénita, refiera cuál de las
padece problemas endocrinológicos y su aumento de peso se debe al buen siguientes complicaciones del tratamiento quirúrgico es más frecuente:
apetito con ingesta consecuente de alimento y la tendencia a la inactividad.
También nos dicen que se acompaña de un aumento de talla importante y de 1. Estenosis traqueal
un adelanto puberal. El índice de masa corporal (IMC) al que hacen referencia 2. Estenosis esofágica
no es correcto, dicho índice se calcula dividiendo el peso (Kg)/ talla2 (cm) y sus 3. Fístula traqueoesofágica recidivante
valores se mueven entre el 18% (desnutrición) y el 40% (obesidad mórbida)
4. Reflujo gastroesofágico
por lo tanto, el 158% al que hace referencia la pregunta no puede correspon-
der a este índice. En realidad se refiere al porcentaje de índice de masa corpo- 5. Traqueomalacia
ral que varía entre 90% (normal) y 120% (obesidad) y que se calcula dividien-
do el IMC del paciente entre el IMC medio para la edad. Teniendo todo esto Esta pregunta corresponde a un examen MIR y se refiere a las posibles compli-
en cuenta es evidente que la niña es obesa y probablemente el aumento de caciones del tratamiento quirúrgico de la fístula traqueoesofágica congénita. La
talla y el adelanto puberal son consecuencia de esta obesidad. Realizaría prue- complicación más frecuente es el reflujo gastroesofágico, como consecuencia
bas hormonales para comprobar que no existe ninguna alteración y, una vez de que al hacer la anastomosis el esfínter puede quedar incompetente.
comprobado, sería suficiente con una restricción calórica para controlar la sin-
tomatología. 66. Señale cuál de las siguientes afirmaciones sobre el reflujo gastroesofágico
NO es correcta:
61. Si aparece alopecia en un niño con raquitismo grave, usted debe pensar en:
1. Consiste en vómitos atónicos o regurgitaciones excesivas durante
1. Déficit carencial de vitamina D las primeras semanas de vida
2. Tubulopatía primaria asociada 2. Resuelven espontáneamente en la mayoría de los casos durante los
3. Déficit de 25hidroxilasa hepática 2 primeros años, aunque algunos persisten hasta los 4 años
4. Déficit de 1-alfa-hidroxilasa renal 3. En la mayoría de los casos hay retraso ponderoestatural
5. Déficit funcional del receptor de vitamina D 4. El diagnóstico es fundamentalmente clínico, la respuesta al trata-
miento confirma el diagnóstico
Esta pregunta también pertenece a un examen MIR, es una pregunta muy con- 5. El tratamiento es fundamentalmente conservador
creta y simplemente hay que recordar que el déficit funcional del receptor de
vitamina D se manifiesta como raquitismo grave con alopecia. En esta pregunta debes reconocer una serie de características sobre el reflujo
gastroesofágico. Recuerda que consiste en vómitos atónicos o regurgitaciones
62. La posición en decúbito prono durante el sueño en los lactantes se ha aso- excesivas durante las primeras semanas de vida y que la mayoría se resuelven
ciado con: espontáneamente cuando el niño empieza a pasar más tiempo en bipedesta-
ción o al espesar las tomas. Lo típico es que no haya retraso ponderoestatural,
1. Una mayor ganancia de peso si este aparece estaríamos hablando de una enfermedad por reflujo gastroeso-
fágico. El diagnóstico es fundamentalmente clínico, cuando se sospecha se
2. Mayor riesgo de neumonía por aspiración
debe iniciar tratamiento y la respuesta a este confirma el diagnóstico. El trata-
3. Disminución en el tránsito intestinal miento es fundamentalmente conservador y solo en raras excepciones se aña-
4. Menor incidencia de vómitos den fármacos o cirugía.
5. Mayor riesgo de muerte súbita
67. Lactante de 21 días de vida que presenta, desde hace 4 días, vómitos tras
Esta pregunta pertenece a un examen MIR y, al contrario que la de la lactancia las tomas que cada vez son más intensos y frecuentes. Señale cuál de las
materna o la del raquitismo, su respuesta es bastante sencilla. Si hay algo que
debes recordar sobre el síndrome de muerte súbita del lactante es que los niños
siguientes afirmaciones es cierta en relación a la enfermedad del niño:
deben dormir en decúbito supino (boca arriba) porque la posición en decúbito
prono durante el sueño en los lactantes se ha asociado con mayor riesgo de 1. Esta enfermedad es más frecuente en varones
muerte súbita. 2. Suele cursar con acidosis metabólica
3. El tratamiento es médico y la recidiva frecuente
63. Dentro de las alteraciones neuropsicológicas están las alteraciones del des- 4. La enfermedad está producida por una bacteria
arrollo, de las cuales la más prevalente es: 5. Es frecuente el antecedente de polihidramnios

1. Incapacidad de aprendizaje Esta es una pregunta de examen MIR y te debe quedar claro el caso clínico por-
2. Retraso mental que siempre lo preguntan igual. Para el MIR, un lactante varón de 21 días de
vida que vomita tiene una estenosis hipertrófica de píloro (EHP) hasta que se
3. Trastorno de déficit de atención con hiperactividad
demuestre lo contrario. El resto de los datos de la pregunta orientan también
4. Parálisis cerebral hacia ese diagnóstico. La respuesta correcta es la primera porque lo típico es
5. Trastorno auditivo que curse con una alcalosis metabólica, ya que gran cantidad del ácido gástri-
co lo pierde con los vómitos. El tratamiento es quirúrgico, se realiza una pilo-
Esta pregunta también pertenece a un examen MIR, es una pregunta directa rotomía que generalmente es definitiva y resuelve la enfermedad. La enferme-
sobre un tema poco importante, simplemente recuerda que dentro de las alte- dad se debe a un crecimiento de la musculatura del píloro y la causa es desco-
raciones neuropsicológicas están las alteraciones del desarrollo, de las cuales la nocida, no se debe a ningún agente infeccioso. En preguntas anteriores hemos
más prevalente es el trastorno de déficit de atención con hiperactividad. mencionado que las obstrucciones digestivas dan lugar a trastornos de la
deglución del líquido amniótico y provocan polihidramnios, pero este no es el
64. En la exploración de un recién nacido usted observa que tiene exceso de caso de la EHP porque aquí la enfermedad no se manifiesta desde el embara-
zo, los recién nacidos son normales y la enfermedad aparece a las 3 semanas
salivación, señale cuál sería su actitud:
de vida.

1. Observaría cuidadosamente la faringe, lo más probable es que


68. Una de las siguientes características NO es típica del estreñimiento funcio-
tenga un absceso retroamigdalino
nal o megacolon funcional:
2. Es frecuente que los recién nacidos tengan abundante salivación,
no le daría importancia
1. Aparece en niños mayores de dos años
3. Intentaría pasar una SNG hasta el estómago
2. No existe distensión abdominal
4. Pediría una radiografía urgente con contraste baritado
3. Suele haber encopresis
5. Colocaría al niño en decúbito supino
4. Suele haber fístulas que sangran con la deposición
5. El tacto rectal muestra una ampolla rectal vacía con tono del esfín-
Esta pregunta nos aporta un único dato que nos tiene que servir para orientar
el cuadro, porque en función de la sospecha clínica tomaremos una actitud ter anal normal
concreta. Nos cuentan que en la exploración de un recién nacido se observa

10 ] PEDIATRÍA [
www.academiamir.com Comentarios test de clase A Mir

Esta pregunta es para diferenciar entre el estreñimiento funcional o megacolon de vaca (APLV) es la causa más frecuente de alergia alimentaria en los lactan-
funcional y el megacolon aganglionar congénito o enfermedad de tes. Dentro de las proteínas de la leche, la implicada suele ser la beta-lactoal-
Hirschprung. Recuerda que en los trastornos funcionales no existen alteracio- búmina, aunque también pueden provocarlo otras (alfa-lactoalbúmina y caseí-
nes morfológicas y se trata más bien de un trastorno psicológico. Los niños na). La diferencia entre APLV e intolerancia a proteínas de leche de vaca (IPLV)
deben tener uso de razón para poder tener trastornos de este estilo, por eso es que en el caso de la APLV suele haber clínica extradigestiva (urticaria, difi-
aparece en mayores de dos años. No suele existir distensión abdominal, aun- cultad respiratoria), mientras que en la IPLV suele ser exclusivamente digestiva.
que como consecuencia de que se aguantan las deposiciones suelen tener fisu- El diagnóstico se basa fundamentalmente en la clínica y las pruebas comple-
ras que sangran con la deposición o se les escapa por rebosamiento (encopre- mentarias (prick cutáneo, rast) sólo ayudan al diagnóstico.
sis). Lo típico del tacto rectal es encontrar una ampolla rectal llena de heces,
con tono del esfínter anal normal. 72. En el diagnóstico de intolerancia por malabsorción de lactosa, todas las
siguientes pruebas son útiles menos:
ENFERMEDAD DE HIRSCHPRUNG MEGACOLON FUNCIONAL
1. Determinación de cuerpos reductores en heces
Desde RN > 2 años 2. Análisis de H2 en aire espirado
3. Test de ureasa en biopsia intestinal
Alteración AP Retención voluntaria 4. Determinación de pH fecal
5. Determinación de ácido láctico en heces
Distensión proximal Distensión de todo el colon
Esta es una pregunta de un examen MIR que quizá te resulte más fácil respon-
Distensión abdominal No distensión abdominal
der cuando hayas estudiado digestivo. Debes recordar que cuando una sus-
Retraso del meconio y tancia no se puede absorber a nivel intestinal, y especialmente si se trata de un
Estreñimiento y encopresis
estreñimiento crónico azúcar, las bacterias intestinales se alimentan de él y fabrican distintas produc-
Se palpan heces en FID tos de degradación (ácidos, H2, etc). Todas las pruebas que te dan en las res-
Se palpan heces en FID
Ampolla rectal con heces puestas son típicas de sustancias de degradación bacterianas a nivel intestinal,
Ampolla rectal vacía
Tono esfínter normal pero el test de la ureasa detecta la presencia de esta encima, que es específica
Tono esfínter normal
Fisuras del Helicobacter pylori.
Posible retraso ponderoestatural No retraso ponderoestatural
73. Indique lo INCORRECTO en relación con la diarrea crónica inespecífica:
Manometría y biopsia alterados Manometría y biopsia normales
1. Es la forma más frecuente de diarrea crónica en la infancia
2. Se presenta entre los 6 meses y los 3 años de edad
69. ¿Cuál es la malformación gastrointestinal congénita más frecuente?
3. Cursa sin alteraciones en el desarrollo ponderal
4. A pesar de la benignidad del proceso, el cuadro suele prolongarse
1. Megacolon aganglionar congénito o enfermedad de Hirschprung
hasta los 8 años
2. Invaginación intestinal
5. Las deposiciones habitualmente contienen moco y restos vegetales
3. Divertículo de Meckel
4. Estenosis hipertrófica de píloro
Esta también es una pregunta que pertenece a un examen MIR en la que se
5. Fístula esofágica refieren a un cuadro bastante frecuente, la diarrea crónica inespecífica. Se con-
sidera que los cólicos del lactante y el colon irritable son variantes de esta
Esta es una pregunta directa, simplemente debes recordar que la malformación misma enfermedad que aparecen en otros momentos de la vida. Es la forma
gastrointestinal congénita más frecuente es el divertículo de Meckel. más frecuente de diarrea crónica en la infancia y se manifiesta como deposi-
ciones con moco y restos vegetales, se cree que como consecuencia de un trán-
70. Una niña de 22 meses, previamente sana, comienza hace 12 horas con un sito gastrointestinal acelerado. Se presenta entre los 6 meses y los 3 años de
cuadro de vómitos y episodios de llanto intermitente, con un estado de dolor edad, momento en el que cede espontáneamente. Es un cuadro benigno que
cursa sin alteraciones en el desarrollo ponderal.
intenso. Coincidiendo con los episodios de dolor, la niña encoge las piernas
hacia el abdomen y se pone pálida y sudorosa. Los padres le enseñan una depo-
sición de color rojo oscuro. El abdomen está distendido y la palpación, aunque 74. Acude a su consulta un niño de 20 meses que desde los 13 meses presenta
difícil, parece resultar dolorosa. Sobre el cuadro que sospecha, señale la afir- un estancamiento de la curva ponderal y disminución del apetito junto con
mación INCORRECTA: deposiciones blandas y abundantes, abdomen distendido y carácter irritable. En
las exploraciones complementarias se observa anemia ferropénica y anticuer-
1. Es la causa más frecuente de obstrucción intestinal entre los 3 pos antiendomisio positivos. Respecto al cuadro clínico citado ¿cuál de las
meses y los 6 años siguientes afirmaciones es FALSA?
2. La localización más frecuente es la ileocólica
3. Aunque el cuadro clínico es muy aparatoso, el estado general del 1. En el diagnóstico es fundamental la realización de una biopsia
niño no suele verse afectado intestinal
4. Si se realiza un tránsito intestinal con contraste baritado es típico 2. Esta enfermedad se desarrolla en personas genéticamente suscep-
el signo del muelle enrollado tibles
5. La ecografía es la prueba diagnóstica de elección 3. El período de presentación clínica más frecuente es el segundo año
de vida, pero no es infrecuente que aparezca en niños más mayores
Esta es una pregunta de MIR que corresponde a un caso clínico, debes grabar- o incluso en la edad adulta
te bien el cuadro porque es la clínica típica de una invaginación intestinal. La 4. La anorexia es un síntoma frecuente
invaginación intestinal es la causa más frecuente de obstrucción intestinal entre 5. La base del tratamiento es la retirada transitoria de la dieta del
los 3 meses y los 6 años y la localización más frecuente es la ileocólica. Se trata trigo, cebada, centeno y avena
de una enfermedad grave y el estado general del niño suele verse muy afecta-
do. La ecografía es la prueba diagnóstica de elección, pero si se realiza un trán- Esta pregunta corresponde a un examen MIR y es importante que te fijes muy
sito intestinal con contraste baritado es típico el signo del muelle enrollado, al bien en ella y sepas reconocer el caso clínico. Es evidente que el niño padece
verse pasar parte del contraste a través de la zona invaginada. una enfermedad celíaca, hay algunos signos y síntomas típicos (estancamiento
de la curva ponderal, disminución del apetito, deposiciones blandas y abun-
71. Con respecto a la intolerancia/alergia a las proteínas de leche de vaca, seña- dantes, abdomen distendido y carácter irritable) y pruebas de laboratorio bas-
le la opción INCORRECTA: tante significativas (anemia ferropénica y anticuerpos antiendomisio positivos).
Para padecer una enfermedad celíaca hay que tener una cierta susceptibilidad
genética y haber estado expuesto al gluten. Lo más frecuente es que se mani-
1. Es la causa más frecuente de alergia alimentaria en los lactantes
fieste durante el segundo año de vida, pero no es infrecuente que aparezca en
2. La proteína implicada es la beta-lactoalbúmina, aunque también niños más mayores o incluso en la edad adulta. La anorexia es un síntoma fre-
pueden provocarlo la alfa-lactoalbúmina y la caseína cuente, es como si los niños supiesen que lo que comen les hace daño. En el
3. La alergia se puede manifestar con clínica digestiva y extradigestiva diagnóstico es fundamental la realización de una biopsia intestinal, ya que aun-
4. La intolerancia se manifiesta a nivel gastrointestinal que se sospeche no se tiene confirmación de la enfermedad hasta no demos-
5. El diagnóstico se basa fundamentalmente en el prick cutáneo trar el daño intestinal. La base del tratamiento es la retirada permanente de la
dieta del trigo, cebada, centeno y avena.
Esta pregunta hace referencia a un tema poco relevante para el MIR, es sufi-
ciente con que tengas algunas ideas generales. La alergia a proteínas de leche 75. La enfermedad celíaca es una intolerancia permanente al gluten ¿Cuál de

11 ] PEDIATRÍA [
www.academiamir.com Comentarios test de clase A Mir

las siguientes afirmaciones es INCORRECTA? Esta pregunta hace referencia a las diarreas agudas, patología bastante fre-
cuente en el niño. El denominado síndrome postgastroenteritis se produce por
1. La lesión intestinal está mediada por mecanismos inmunológicos retrasar la nutrición durante el episodio diarreico, la lactancia materna protege
frente a su aparición por lo que no se debe retrasar la ingesta de leche. La
2. Suele existir un intervalo libre de síntomas entre la introducción del
mayor parte de las veces no se aísla el agente causal de la diarrea y lo que se
gluten en la dieta y el comienzo de la clínica observa es que las deposiciones tienen más grasa de lo normal.
3. La determinación de anticuerpos antigliadina es la prueba sexoló-
gica más específica de esta enfermedad 79. Acude a urgencias de su hospital un niño de 4 años con un cuadro de difi-
4. La infestación por G. lamblia puede dar un cuadro clínico similar cultad respiratoria, ronquera, tos perruna y estridor inspiratorio. Los padres
5. El diagnóstico de la enfermedad se basa en la demostración de una refieren que tiene febrícula desde hace varios días y que la dificultad respirato-
lesión en la mucosa intestinal ria empezó a primera hora de la noche y ha ido en aumento en las últimas
horas. En la exploración física usted observa leve tiraje intercostal, aleteo nasal
Esta pregunta pertenece a un examen MIR. Nos preguntan por el diagnóstico
de la enfermedad celíaca, cuya lesión intestinal está mediada por mecanismos
y estridor inspiratorio en la auscultación. Señale cual de las siguientes opciones
inmunológicos y suele existir un intervalo libre de síntomas entre la introduc- le parece INCORRECTA:
ción del gluten en la dieta y el comienzo de la clínica. La determinación de anti-
cuerpos antitransglutaminasa es la prueba serológica más específica de esta 1. Se debe realizar una radiografía de faringe para hacer el diagnós-
enfermedad y los anticuerpos antigliadina pueden ser positivos en muchas tico diferencial del cuadro
otras enfermedades, por ejemplo la infestación por G. lamblia (que puede dar 2. Se deben visualizar las amígdalas, de manera directa mediante el
un cuadro clínico similar). Para confirmar el diagnóstico de la enfermedad, aun- uso de un depresor, para descartar la presencia de exudados
que se sospeche por la clínica y los anticuerpos, precisa de la demostración de
3. El tratamiento es fundamentalmente sintomático (oxigenoterapia,
una lesión anatomopatológica en la mucosa intestinal.
corticoides, adrenalina, etc) y el ambiente húmedo y frío disminuye la
sintomatología
76. El síndrome de Reye se ha asociado con uno de los siguientes fármacos,
4. En casos muy graves el paciente puede precisar intubación endo-
señale con cuál de ellos:
traqueal
5. No se deben usar antitusígenos, antihistamínicos ni sedantes
1. Furosemida
2. Metotrexate
Esta pregunta es un caso clínico y de nuevo, lo primero que tienes que hacer
3. Amoxicilina es reconocer la enfermedad. La clínica que nos cuentan corresponde a un crup,
4. Ácido acetil salicílico la duda sería si se trata de una laringotraqueobronquitis o una laringitis aguda.
5. Heparina En cualquier caso, lo que nunca se debe hacer es intentar visualizar la faringe,
ya que existe el riesgo de que se produzca un espasmo laríngeo que produci-
Esta es una pregunta directa que cayó en un examen MIR, de este síndrome ría la muerte del paciente. El resto de las preguntas son ciertas, se debe reali-
todo lo que debéis recordar es que se asocia al ácido acetil salicílico y a ciertas zar una radiografía de faringe para hacer el diagnóstico diferencial con patolo-
infecciones víricas (varicela, gripe). Se manifiesta como un daño hepático y a gías más graves como la epiglotitis aguda (siempre que el estado clínico del
nivel del sistema nervioso central y es el causante de que hayan retirado de las paciente lo permita). El tratamiento es fundamentalmente sintomático (oxige-
farmacias las aspirinas infantiles que tomábamos de pequeños, que sabían a noterapia, corticoides, adrenalina, etc) y el ambiente húmedo y frío disminuye
naranja y que estaban tan buenas. la sintomatología, aunque en casos muy graves el paciente puede precisar intu-
bación endotraqueal. No se deben usar antitusígenos, antihistamínicos ni
sedantes que puedan disminuir el esfuerzo respiratorio del paciente.
77. Señale cuál de las siguientes afirmaciones es INCORRECTA con respecto a
las deshidrataciones:
80. Acude a urgencias de su hospital un niño de 2 años que inmigrante y vive
1. Dado que los lactantes tienen más proporción de agua en su orga- en un barrio marginal de su ciudad. Desde hace aproximadamente una hora
nismo pueden perder mayor cantidad de agua que un niño mayor y presenta un cuadro de dificultad respiratoria coincidente con fiebre alta y mal
verse menos afectado su estado clínico estado general. Usted observa que el niño no llora, mantiene una postura de
2. En las deshidrataciones hipertónicas existe mayor riesgo de shock hiperextensión del cuello con la boca abierta y babeo y presenta importante
hipovolémico que en las deshidrataciones hipotónicas tiraje intercostal, supraesternal y aleteo nasal. Señale cual sería la opción INCO-
3. En el hemograma se puede ver una hemoconcentración con RRECTA:
aumento de hematocrito, hemoglobina y proteínas plasmáticas
4. La mejor forma de evaluar a corto plazo la respuesta al tratamien- 1. El cuadro puede corresponder a un proceso muy grave y mortal
to en un niño deshidratado es midiendo la diuresis que ocurre en niños de 2 a 7 años
5. La reposición de una deshidratación hipernatrémica debe hacerse 2. Normalmente no hay un ambiente familiar de infección
mucho más lentamente que una hiponatrémica, para evitar el posi- 3. La incidencia ha ido en aumento dado que no existe una vacuna
ble edema cerebral completamente eficaz en el mercado
4. Nunca se debe explorar la faringe en casos dudosos, si es preciso
En esta pregunta se hace referencia a algunas características de las deshidrata- se realizará en quirófano con laringoscopia directa
ciones. Recuerda que los lactantes tienen más proporción de agua en su orga- 5. El tratamiento de elección son los antibióticos parenterales duran-
nismo y pueden “permitirse el lujo” de perder un poco más de agua. Así, si un te 10 días (cefotaxima, ceftriaxona o ampicilina) junto con intubación
lactante podría perder hasta un 8% de su peso y se seguiría considerando una si precisa
deshidratación moderada mientras que en el niño mayor sería considerada una
deshidratación grave. En las deshidrataciones hipertónicas las células ceden su Esta pregunta sirve para recordar que en el MIR siempre que dan un dato es
agua al espacio extravascular y de esta forma lo protegen frente al shock hipo- por algo. En este caso nos dicen que el niño es inmigrante y vive en un barrio
volémico, que por otro lado es típico de las deshidrataciones hipotónicas. En marginal, lo que es sinónimo de decir que seguramente estará mal vacunado.
las deshidrataciones podemos observar en el hemograma signos de hemocon- Presenta un cuadro que se caracteriza fundamentalmente por clínica respirato-
centración (aumento de hematocrito, hemoglobina y proteínas plasmáticas). La ria grave (no llora, mantiene una postura de hiperextensión del cuello con la
mejor forma de evaluar a corto plazo la respuesta al tratamiento en un niño boca abierta y babeo y presenta importante tiraje intercostal, supraesternal y
deshidratado es midiendo la diuresis, que por otro lado es uno de los primeros aleteo nasal) coincidente con fiebre alta y mal estado general. El cuadro clínico
síntomas que aparecen cuando un niño se deshidrata. La reposición de una corresponde a una epiglotitis aguda, proceso muy grave y mortal que ocurre
deshidratación hipernatrémica debe hacerse mucho más lentamente que una en niños de 2 a 7 años. La incidencia ha disminuido desde la vacunación siste-
hiponatrémica, para evitar el posible edema cerebral. mática frente al agente causal (H. influenzae). Normalmente no hay un
ambiente familiar de infección, a diferencia de otros cuadros como la traqueo-
78. ¿Cuál de las siguientes opciones le parece la más correcta? bronquitis. Como en la pregunta anterior, nunca se debe explorar la faringe en
casos dudosos, si es preciso se realizará en quirófano con laringoscopia direc-
1. En el síndrome postgastroenteritis, la mayor parte de las veces se ta. Dado que se trata de un cuadro muy grave, el tratamiento de elección son
los antibióticos parenterales durante 10 días (cefotaxima, ceftriaxona o ampici-
aisla el agente causal de la diarrea
lina) junto con intubación si precisa.
2. El análisis bioquímico de las heces es normal
3. Se debe retrasar la nutrición hasta que cese el episodio diarreico 81. Una madre le consulta porque su niño de 20 días de vida presenta casi
4. La lactancia materna protege frente a la aparición de este síndrome desde el nacimiento un estridor inspiratorio de forma intermitente, especial-
5. En niños con diarrea importante se debe retrasar la ingesta de mente cuando está en decúbito supino. No se acompaña de dificultad respira-
leche por lo menos 15 días tras el episodio toria ni ninguna otra sintomatología. ¿Cuál es su diagnóstico?

12 ] PEDIATRÍA [
www.academiamir.com Comentarios test de clase A Mir

1. Epiglotitis aguda cuál es la opción correcta:


2. Laringitis aguda
3. Traqueobronquitis aguda 1. El síntoma más frecuente a nivel respiratorio es la tos seca que
4. Laringomalacia empeora con la edad, por las mañanas y con el esfuerzo
5. Bronquiolitis obliterante 2. Las sobreinfecciones respiratorias son frecuentes, especialmente
por micobacterias
Esta pregunta hace referencia a la clínica típica de una laringomalacia. 3. No suelen tener clínica de sinusitis aunque es frecuente encontrar
Simplemente debes recordar que se trata de una inmadurez de los cartílagos los senos opacificados en las radiografías
laríngeos y traqueales y que es un cuadro leve que desaparece con la madura- 4. No suele haber alteraciones de vía aérea superior, la clínica respi-
ción de dichos cartílagos.
ratoria es exclusivamente de vías respiratorias bajas
5. Es raro que aparezcan manifestaciones de enfermedad pulmonar
82. Con respecto a la bronquiolitis aguda, señale la opción INCORRECTA: crónica como atelectasias, hemoptisis, neumotórax, osteoartropatía
hipertrófica, etc.
1. La etiología suele ser vírica y en la mayor parte de los casos se debe
al VRS Esta pregunta también se refiere a la fibrosis quística, en este caso a la clínica.
2. Los niños mayores y adultos, aunque se contagien no desarrollan Es cierto que el síntoma más frecuente a nivel respiratorio es la tos pero no es
la enfermedad seca sino productiva. Las sobreinfecciones respiratorias son frecuentes, peor
3. Es más frecuente que aparezca en ambientes familiares de fuma- generalmente se infectan por bacterias bastante frecuentes en la población (S.
dores o enfermedad respiratoria poco importante aureus, P. aeruginosa, etc) no por micobacterias. Suelen tener afectación de las
4. En la radiografía de tórax se pueden ver tanto signos de hiperin- vías respiratorias superiores (por ejemplo pólipos) y, aunque no suelen tener clí-
nica de sinusitis, es frecuente encontrar los senos opacificados en las radiogra-
suflación como de atelectasia
fías. Es muy frecuente que aparezcan manifestaciones de enfermedad pulmo-
5. El tratamiento de elección es la ribavirina nar crónica como atelectasias, hemoptisis, neumotórax, osteoartropatía hiper-
trófica, etc.
Esta pregunta hace referencia a una de las patologías más frecuentes de los
niños, la bronquiolitis aguda. La etiología de este cuadro suele ser vírica y en la
mayor parte de los casos se debe al virus respiratorio sincitial (VRS). Los niños
86. Señale cuál de las siguientes complicaciones no es típico que aparezca en
mayores y adultos que se contagian no desarrollan una enfermedad de vías res- la fibrosis quística:
piratorias pequeñas (que es lo típico) sino simplemente un catarro de vías altas.
Es más frecuente que aparezca en ambientes familiares de fumadores o enfer- 1. Íleo meconial
medad respiratoria poco importante. En la radiografía de tórax se pueden ver 2. Cirrosis biliar
tanto signos de hiperinsuflación como de atelectasia, como consecuencia de 3. Cetoacidosis diabética
los tapones de moco en la pequeña vía aérea (bronquiolos). El tratamiento es 4. Azoospermia
exclusivamente sintomático (oxigenoterapia, fisioterapia respiratoria, aspiracio-
5. Cervicitis
nes frecuentes, etc), la ribavirina es un fármaco antiguo que está actualmente
en desuso por la cantidad de efectos secundarios que tiene (muy teratógena,
incluso para el personal sanitario que trabaja en las unidades en las que se Esta pregunta se refiere a las complicaciones de la fibrosis quística. Recuerda
administra) y porque realmente no es demasiado efectiva. que a partir de los 20 años de evolución de la enfermedad se puede ver afec-
tada la parte endocrina del páncreas (islotes pancreáticos) y aparecer una dia-
betes mellitus, pero es excepcional que ésta se manifieste en forma de cetoa-
83. Señale cuál de las opciones siguientes sobre la bronquiolitis obliterante es cidosis diabética. El resto de las complicaciones mencionadas en la pregunta
verdadera: pueden aparecer en los enfermos de fibrosis quística en distintos momentos de
su vida.
1. La causa fundamental es una infección vírica
2. Con frecuencia se relaciona con el trasplante pulmonar y el de 87. Señale cuál de los siguientes NO se considera criterio diagnóstico de fibro-
médula ósea sis quística:
3. La enfermedad se controla en pocos días con broncodilatadores
inhalados 1. Prueba del sudor positiva
4. El diagnóstico se realiza mediante radiografía de tórax en la que se 2. Enfermedad pulmonar crónica
observa un patrón muy característico 3. Insuficiencia pancreática endocrina
5. El cuadro se resuelve por completo y no suele haber recidivas 4. Antecedentes familiares positivos
5. Test de screening neonatal positivo
Esta pregunta es fundamentalmente para destacar que la bronquiolitis oblite-
rante no tiene nada que ver con la bronquiolitis aguda. Lo más importante que En esta pregunta se pregunta por los criterios diagnósticos de fibrosis quística.
debes recordar es la asociación de esta enfermedad con el trasplante pulmonar De todas las opciones que nos dan, la única que no es criterio diagnóstico de
y el de médula ósea y la dificultad para tratarla y controlarla. FQ es la insuficiencia pancreática endocrina, lo típico es la afectación exocrina,
la endocrina se produce a partir de los 20 años de evolución de la enfermedad.
84. Señale cual de las siguientes opciones es INCORRECTA sobre la fibrosis
quística: 88. Señale cuál de las siguientes afirmaciones sobre el tratamiento de la fibro-
sis quística es INCORRECTO:
1. Es la enfermedad genética más frecuente y grave de la raza cau-
cásica, con herencia autosómico dominante 1. Uno de los pilares fundamentales en el tratamiento es la fisioterá-
2. Es la causa más frecuente de enfermedad pulmonar grave en la pia respiratoria
infancia 2. Los antibióticos se usan en dosis elevadas y controlan la progresión
3. La alteración se produce en el gen que codifica una proteína regu- de la enfermedad
ladora transmembrana 3. Los corticoides en dosis altas se utilizan cuando existe hiperreacti-
4. Consiste en una disfunción de glándulas exocrinas con secreciones vidad refractaria de las vías aéreas
espesas que ocasionan obstrucción 4. En las atelectasias debe valorarse la lobectomía si hay dificultad
5. Las manifestaciones clínicas más importantes se producen a nivel respiratoria progresiva
de las vías aéreas y del aparato digestivo 5. La hemoptisis masiva requiere vitamina K si el tiempo de protrom-
bina está alterado, y suspensión de la fisioterapia
Esta pregunta hace referencia a la fibrosis quística, dado que se trata de una
enfermedad muy importante y frecuente, es necesario saberlo casi todo. Es la Esta pregunta se refiere al tratamiento de la fibrosis quística, en el que insisti-
enfermedad genética más frecuente y grave de la raza caucásica, con herencia remos en la fase de consolidación. Es importantísimo que estos pacientes
autosómico recesiva. Es la causa más frecuente de enfermedad pulmonar grave hagan una buena fisioterapia respiratoria para eliminar las secreciones, es uno
en la infancia. La alteración se produce en el gen que codifica una proteína de los pilares fundamentales en el tratamiento. Los antibióticos se usan en
reguladora transmembrana, por lo que las secreciones serán más espesas de lo dosis elevadas y controlan la progresión de la enfermedad, desde el momento
normal y se producirá una disfunción de glándulas exocrinas, con la obstruc- en el que empieza una infección respiratoria se debe tratar de manera enérgi-
ción consecuente. Las manifestaciones clínicas más importantes se producen a ca. Los corticoides, por el contrario, se deben usar en dosis bajas, cuando exis-
nivel de las vías aéreas y del aparato digestivo. te hiperreactividad refractaria de las vías aéreas. En las atelectasias debe valo-
rarse la lobectomía si hay dificultad respiratoria progresiva. La hemoptisis masi-
85. Con respecto a las manifestaciones clínicas de la fibrosis quística, señale va requiere vitamina K si el tiempo de protrombina está alterado, y suspensión

13 ] PEDIATRÍA [
www.academiamir.com Comentarios test de clase A Mir

de la fisioterapia, ya que ésta podría favorecer el sangrado. de testosterona no se detecta es porque los testículos no están o son gónadas
atróficas que no funcionan. Los testículos están ubicados en la bolsa escrotal
89. Las infecciones urinarias: porque las altas temperaturas del abdomen pueden provocar daño en las
gónadas masculinas (esterilidad, aparición de tumores, etc).
1. Son más frecuentes en varones
2. Son factor de riesgo para el desarrollo de insuficiencia renal y 93. Ante la sospecha de torsión testicular, ¿cuál de las siguientes es la prueba
nefropatía terminal diagnóstica de elección?
3. En las niñas, el agente más frecuente es el E. coli
4. Las bacteriurias asintomáticas en los niños requieren tratamiento 1. Ecografía doppler
como si se tratase de infecciones 2. Gammagrafía isotópica
5. El hecho de que exista febrícula en una ITU baja debe hacer pen- 3. Ecografía
sar que evoluciona hacia una PNA 4. Radiografía escrotal
5. Palpación bimanual
Esta pregunta hace referencia a las infecciones del tracto urinario (ITUs). Son
una patología más frecuente en las mujeres (porque tienen la uretra más corta Esta pregunta pertenece a un examen MIR. La torsión testicular es uno de los
que los varones y es más fácil su colonización desde la zona anal) salvo en el cuadros más graves que se pueden producir a nivel genitourinario, ya que un
primer año de la vida, que son más frecuentes en los niños (por la fimosis fisio- diagnóstico tardío puede acabar en la necrosis del testículo y la pérdida de la
lógica que favorece que se acumulen los gérmenes). Es muy dudoso si las ITUs gónada. La prueba diagnóstica de elección es la ecografía doppler en la que se
son o no factor de riesgo para el desarrollo de insuficiencia renal y nefropatía vería si la vascularización del testículo está íntegra.
terminal, ya que son una patología muy frecuente entre los niños y realmente
son muy pocos los que desarrollan una insuficiencia renal. En las niñas, el agen- 94. Señale cuál de las siguientes afirmaciones sobre patología testicular, del
te más frecuente es el E. coli y en los niños se aíslan por igual el E. coli y el pene y de la uretra le parece INCORRECTA:
Proteus. Las bacteriurias asintomáticas no se tratan en los niños, únicamente se
administra tratamiento en las mujeres embarazadas porque pueden provocar
1. La torsión de hidátide es la causa más frecuente de dolor testicu-
pielonefritis agudas (PNA). Algunas ITUs bajas (cistitis) pueden tener febrícula,
sin que exista necesariamente una PNA.
lar en niños de 2 a 11 años
2. Una epididimitis es una infección del epidídimo, generalmente por
90. Señale la causa más frecuente de dilatación renal en la infancia: vía hematógena, que puede afectar o no al testículo
3. En el hidrocele se produce una acumulación de líquido en la túnica
1. Litiasis vaginal que provoca tumefacción testicular no dolorosa homogénea
2. Infecciones urinarias 4. En las epispadias la uretra está situada en la parte superior del
3. Tumor de Wilms pene
4. Reflujo vesicoureteral 5. En las hipospadias proximales asociadas a criptorquidia debe estu-
5. Quistes renales diarse el cariotipo

Esta pregunta es un resumen de algunas de las patologías urológicas más fre-


Esta es una pregunta directa que pertenece a un examen MIR. La causa más
cuentes en los niños. Recuerda que la causa más frecuente de dolor testicular
frecuente de dilatación renal en la infancia es el reflujo vesicoureteral.
en niños de 2 a 11 años es la torsión de hidátide. La infección del epidídimo o
epididimitis se produce generalmente por vía retrógrada desde la uretra y
91. Señale cuál de las siguientes afirmaciones referentes al reflujo vesicourete- puede afectar o no al testículo. En el hidrocele se produce una acumulación de
ral grado I en el niño es INCORRECTA: líquido en la túnica vaginal que provoca tumefacción testicular no dolorosa
homogénea. En las epispadias el orificio uretral está situado en la parte supe-
1. Se diagnostica por cistografía rior del pene o porción dorsal de este (considerando el pene en erección). En
2. Se debe tratar con profilaxis antibiótica las hipospadias la uretra se encuentra en posición ventral (de nuevo con el pene
en erección) y en el caso de las proximales asociadas a criptorquidia debe estu-
3. Tiene tendencia a desaparecer
diarse el cariotipo para descartar hermafroditismo y otros síndromes.
4. No produce dilatación de la pelvis y cálices renales
5. En un niño menor de un año debe corregirse quirúrgicamente de
95. Niña de 3 años que presenta afectación brusca del estado general y pre-
forma precoz
sencia de petequias puntiformes diseminadas. La orina es hematúrica y se cons-
Esta pregunta pertenece a un examen MIR y en ella se hace referencia al reflu-
tata hipertensión arterial. El hemograma muestra Hb7 g/dl; 17000 leucocitos
jo vesicoureteral grado I. El reflujo de grado I es aquel en el que la orina no con neutrofilia, 37000 plaquetas con normalidad de las pruebas de coagula-
refluye la totalidad del uréter y no produce dilatación de la pelvis ni de los cáli- ción. ¿Cuál es el diagnóstico más probable?
ces renales. Es cierto que el reflujo, de cualquier grado, se diagnostica por cis-
tografía (introducción de contraste por la uretra para ver la competencia del 1. Sepsis por Salmonella
esfínter vesicoureteral). La mayoría de los de grado I y II y algunos de grado III 2. Púrpura de Schönlein-Henoch
desaparecen espontáneamente, por lo que no es necesario intervenir en la 3. Glomerulonefritis aguda post-infecciosa
mayoría de los casos. Mientras se espera la reversibilidad del cuadro, se debe
4. Síndrome hemolítico-urémico
tratar con profilaxis antibiótica al niño para que la orina que refluye hacia los
riñones sea lo más estéril posible.
5. Coagulación intravascular diseminada

Esta pregunta corresponde a un examen MIR y el cuadro al que hace referen-


92. Sobre la criptorquidia, señale la afirmación INCORRECTA: cia se explica de forma más detallada en nefrología. La niña padece un síndro-
me hemolítico urémico, donde lo típico es que haya tenido en los días previos
1. Es el trastorno más frecuente de la diferenciación sexual en los algún tipo de infección y, como consecuencia de ésta, se produce una afecta-
niños ción a nivel de los pequeños vasos, especialmente renales (insuficiencia renal
2. Es más frecuente en niños prematuros aguda). La afectación vascular provoca la aparición de trombos que favorecen
3. La hormona antimulleriana está baja y tras administrar hCG no se la destrucción de los hematíes (anemia hemolítica) y de las plaquetas (trombo-
d tecta aumento de testosterona penias, por lo que se producen petequias).
4. La mayoría descenderán de forma espontánea hacia los 3 meses
de vida 96. En un niño de 18 meses, previamente sano, descubre usted una masa tumo-
5. Algunas de sus complicaciones son la esterilidad y la aparición de ral de localización presacra. Antes de realizar pruebas complementarias, seña-
tumores, especialmente en aquellos testículos que permanecen le en qué diagnóstico pensaría en primer lugar:
durante mucho tiempo en la cavidad abdominal
1. Neuroblastoma
Esta pregunta se refiere al trastorno más frecuente de la diferenciación sexual 2. Nefroblastoma
en los niños: la criptorquidia o testículo oculto. El desarrollo testicular se pro- 3. Rabdomiosarcoma
duce en el abdomen y durante las últimas semanas del embarazo los testículos 4. Mesenquimoma
descienden al escroto, por eso algunos niños, especialmente los prematuros no 5. Teratoma maligno
nacen con los testículos en la bolsa escrotal. La mayoría de estos testículos ocul-
tos descenderán de forma espontánea hacia los 3 meses de vida. Cuando los
Esta fue también una pregunta de MIR. Se trata de una pregunta directa en la
testículos no son palpables en el canal escrotal, se debe descartar que estos
que el dato clave es la localización del tumor. Recuerda que cuando nos hablen
estén ausentes, para ello se administra al niño hCG y si los testículos están pre-
de masa tumoral de localización presacra hay que pensar en primer lugar en un
sentes fabricarán testosterona que se detectará en plasma. Si dicho aumento

14 ] PEDIATRÍA [
www.academiamir.com Comentarios test de clase A Mir

teratoma maligno. es un tumor que afecta al riñón, es raro que se produzca hematuria. Única-
mente se puede hablar de curación si se extirpa el tumor. Aunque lo más fre-
97. Acude a urgencias de su hospital un niño de 4 años con un cuadro de vómi- cuente es que no se afecten ambos riñones, existen ciertos síndromes que cur-
san con frecuencia con tumores de Wilms bilaterales (síndromes de WAGR, de
tos de predominio matutino, cefalea, alteraciones visuales e hipotalámicas. Denys-Drarh, de Beckwith-Wiedemann, etc). Las metástasis más frecuentes son
Aporta una radiografía lateral de cráneo en la que se observan calcificaciones las pulmonares (las óseas son poco frecuentes) que pueden tratarse con éxito.
en forma de paréntesis en la zona supraselar. ¿Cuál sería su diagnóstico?
101. Con respecto al diagnóstico y tratamiento del nefroblastoma, señale la
1. Glioma del nervio óptico opción INCORRECTA:
2. Craneofaringioma
3. Meduloblastoma 1. La primera prueba a realizar debe ser la ecografía
4. Pinealoma productor de hidrocefalia 2. Si existen dudas diagnósticas se debe biopsiar el tumor
5. Adenoma hipofisario 3. La TAC muestra una masa no homogénea con zonas de necrosis
4. La quimioterapia preoperatoria está indicada en tumores bilaterales
Esta pregunta corresponde a un examen MIR y en ella se observan signos de
5. El pronóstico es mejor en niños menores de 2 años
hipertensión intracraneal (vómitos de predominio matutino, cefalea) y altera-
ciones visuales e hipotalámicas (que indican que el tumor se sitúa en la región
central del cráneo). Además nos aportan un dato que permite hacer el diag- Esta pregunta se refiere también a tumores en el niño. En el caso de los tumo-
nóstico de certeza, en la radiografía lateral de cráneo se observan calcificacio- res abdominales, la primera prueba a realizar debe ser la ecografía (siempre
nes en forma de paréntesis en la zona supraselar, que son muy sugestivas de que se pueda se debe intentar evitar radiar al niño, por lo que lo ideal es empe-
craneofaringioma. zar el diagnóstico con una ecografía). El estudio de extensión se hace median-
te TAC, que muestra una masa no homogénea con zonas de necrosis. Si exis-
ten dudas diagnósticas no se debe biopsiar el tumor porque esto modifica el
98. Niña de 18 meses que es remitida aa su hospital porque le detectan en su estadiaje (se rompe la cápsula y cambia el estadio), como mucho se podría
centro de salud una masa abdominal. Usted le palpa una masa dura en fosa hacer una punción-aspiración con aguja fina (PAAF). La quimioterapia preope-
renal derecha y le observa un mínimo exoftalmos derecho con hematoma line- ratoria está indicada en los tumores bilaterales. En general el pronóstico es
al en párpado superior derecho. El resto de la exploración no es significativa. mejor en niños menores de 2 años.
Entre los exámenes complementarios destacan niveles elevados de ferritina y
catecolaminas en orina. ¿Qué sospecha que tiene la niña? 102. Señale cuál de las siguientes afirmaciones le parece INCORRECTA con res-
pecto al sarampión:
1. Un neuroblastoma
2. Un tumor de Wilms 1. Se ha observado últimamente un nuevo brote en jóvenes que esca-
3. Un nefroma mesoblástico paron a la vacunación sistemática
4. Un teratoma 2. Una vez que aparece el exantema no hay peligro de contagio
5. Un hipernefroma 3. El exantema afecta característicamente a palmas y plantas
4. Es la enfermedad exantemática que con mayor frecuencia da
De nuevo una pregunta que corresponde a un examen MIR, en este caso se manifestaciones neurológicas
refiere a un tumor abdominal. El hematoma lineal en párpado hace sospechar 5. Existe una profilaxis pasiva con gammaglobulina específica y otra
que estamos ante un neuroblastoma. activa mediante vacunación

99. Acude a su consulta un lactante de 10 meses de edad para un control ruti- Con esta pregunta comienzan las enfermedades infecciosas y dentro de estas
nario y usted le descubre una masa en flanco izquierdo, dura, que sobrepasa las exantemáticas. Con respecto al sarampión debes recordar que hasta hace
línea media. En la ecografía abdominal dicha masa se corresponde con un 26 años se ponía una sola dosis de vacuna, que no ha resultado efectiva en
tumor sólido localizado en la glándula suprarrenal izquierda. ¿Cuál de las todos los casos y por lo tanto parte de la población no se encuentra inmuniza-
siguientes afirmaciones es cierta? da correctamente (esto ha provocado que últimamente haya aparecido un
nuevo brote en jóvenes). El sarampión se contagia desde la fase prodrómica
hasta 5 días después de la aparición de este y es la enfermedad exantemática
1. Si presenta metástasis hepáticas estará contraindicado el trata- que con mayor frecuencia da manifestaciones neurológicas. Dicho exantema
miento quirúrgico afecta característicamente a palmas y plantas. Existe una profilaxis pasiva con
2. La edad menor de un año empeora el pronóstico gammaglobulina específica y otra activa mediante vacunación.
3. Debe realizarse aspirado de médula ósea como parte del estudio
de extensión 103. Una lactante de un año de edad presenta fiebre de 24 horas de evolución,
4. El tumor que presenta es más frecuente en pacientes con hemihi- cuadro catarral nasal y un exantema generalizado, rosado y poco confluente. En
pertrofia la exploración se palpan adenopatías numerosas de predominio occipital. El
5. El empleo de la gammagrafía con metayodobencilguanidina ha diagnóstico más probable es:
sido abandonada por técnicas más específicas
1. Rubéola
Esta pregunta también pertenece a un examen MIR y en ella se hace referencia 2. Sarampión
a un neuroblastoma (que es el tumor que típicamente vamos a encontrar en un 3. Mononucleosis infecciosa
niño con un tumor sólido localizado en la glándula suprarrenal). Las metástasis
4. Exantema súbito
no contraindican el tratamiento quirúrgico, de hecho hay ciertas metástasis que
condicionan mejor pronóstico que otras y en las que se han descrito hasta remi- 5. Escarlatina
siones espontáneas (recuerda el estadio IVs que se produce en niños menores
de un año, con tumor primario y diseminación a hígado, piel o médula ósea). La Esta pregunta corresponde a un examen MIR. Fíjate en las características espe-
edad menor de un año no empeora el pronóstico, ya hemos dicho que en algu- cíficas de cada enfermedad exantemática para poder diferenciar unas de otras,
nos casos se han descrito incluso remisiones espontáneas. Uno de los lugares en el caso de la rubéola (que es la enfermedad a la que hace referencia la pre-
más frecuentes de metástasis es la médula ósea, por eso se debe incluir el aspi- gunta) el dato típico son las adenopatías de predominio occipital.
rado medular como parte del estudio de extensión. Para el diagnóstico se sigue
utilizando la gammagrafía con metayodobencilguanidina. El tumor que se aso- 104. Acude a la urgencia de su hospital un niño de 10 años que refiere cefalea,
cia con frecuencia a la hemihipertrofia es el nefroblastoma o tumor de Wilms y escalofríos, fiebre y odinofagia. En la exploración usted observa que la lengua
no el neuroblastoma que es el que padece nuestra paciente.
es muy roja y que presenta exudados amigdalares, además empieza a aparecer
un exantema palpable (rasposo) más marcado en las flexuras. Señale la opción
100. Señale cuál de las siguientes afirmaciones sobre el tumor de Wilms o correcta:
nefroblastoma es INCORRECTA:
1. Lo más probable es que el niño no esté vacunado y por eso sufre
1. La hematuria es rara la enfermedad
2. Únicamente se cura si se extirpa el tumor 2. El tratamiento sería fundamentalmente sintomático
3. No afecta a ambos riñones 3. Se pueden hacer test rápidos de detección del patógeno en faringe
4. Las metástasis óseas son poco frecuentes 4. No aparecen complicaciones por extensión local del agente
5. Las metástasis pulmonares pueden tratarse con éxito 5. En algunas ocasiones está indicado el tratamiento antibiótico
Esta pregunta se refiere al otro tumor infantil que hay que conocer para el
Esta pregunta es un caso clínico que refleja claramente las características típi-
MIR, también pertenece a un examen de años previos. Recuerda que aunque

15 ] PEDIATRÍA [
www.academiamir.com Comentarios test de clase A Mir

cas de una escarlatina (lengua aframbuesada, exudados amigdalares y exante- el agente causal de la enfermedad es el VH6, no suele tener ninguna otra sin-
ma palpable o rasposo más marcado en las flexuras). El agente causal de esta tomatología (salvo crisis febriles en alguna ocasión coincidiendo con la etapa
enfermedad es el Streptococcus piógenes o ß-hemolítico grupo A (SBHGA) y febril), el exantema desaparece en pocos días y no es necesario dar ningún tra-
no existe vacuna frente a él. Existen test rápidos que mediante un frotis farín- tamiento.
geo nos pueden decir en minutos si el bicho se encuentra presente en los exu-
dados amigdalares. Cuando diagnosticamos la enfermedad, para evitar las 108. Le consultan porque un niño que está ingresado en la planta de pediatría
posibles complicaciones asociadas a la infección (glomerulonefritis, fiebre reu-
mática), es necesario administrar tratamiento antibiótico (penicilina). En algu-
por fiebre de varios días de evolución presenta afectación del estado general y
nas ocasiones se pueden producir complicaciones a nivel local (por ejemplo ligera irritabilidad. En la exploración usted observa un exantema maculopapu-
abscesos periamigdalinos). loso discreto en tronco e hiperemia conjuntival bilateral sin secreción. Presenta
lengua aframbuesada y enrojecimiento bucal sin exudados amigdalares y ade-
105. La madre de una compañera del colegio de su hija de 6 años le consulta nopatías cervicales, rodaderas de aproximadamente 1 cm. Su pediatra le había
porque la niña presenta desde hace 4 días lesiones cutáneas que inicalmente pautado azitromicina (3 dosis). Señale cuál de los siguientes le parece el diag-
eran rosadas y posteriormente se transformaron en unas vesículas con conteni- nóstico más probable:
do transparente en tronco, cara y brazos. Algunas de ellas ha evolucionado a
costras. Usted le "echa un vistazo" a la niña y observa que tiene fiebre de 38ºC 1. Escarlatina
y que, ademas de las lesiones vesículo-pustulosas descritas, presenta en el velo 2. Rubéola
del paladar 5 lesiones ulcerosas de 2 mm de diámetro.¿Qué le diría a la madre? 3. Exantema súbito
4. Mononucleosis infecciosa
1. Que su hija padece una primoinfección herpética y que puede 5. Síndrome de Kawasaki
seguir acudiendo al colegio con normalidad
2. Que la niña padece un síndrome de mano-pie-boca y que debe Esta pregunta resume varias de las características de la enfermedad de
Kawasaki (fiebre de origen desconocido, exantema maculopapuloso, hiperemia
acudir a un dermatólogo para tratarlo correctamente
conjuntival bilateral sin secreción, lengua aframbuesada, enrojecimiento bucal
3. Que la niña tiene un impétigo infeccioso y le recomienda un tra- sin exudados amigdalares y adenopatías cervicales), únicamente faltaría el
tamiento antibiótico con cloxacilina edema de manos y pies para completar todos los criterios clínicos de la enfer-
4. Que su hija padece una varicela y que lo mejor será que se quede medad. El pediatra sospechó una infección de tipo bacteriano (probablemente
en casa hasta que todas las lesiones estén en fase de costra escarlatina, aunque no pega lo de la conjuntivitis) y trató con un antibiótico que
5. Que su hija tiene probablemente una erupción variceliforme de no ha hecho remitir el cuadro. Recuerda que el tratamiento, cuando se sospe-
Kaposi cha la enfermedad, son las inmunoglobulinas (y que no se deben usar los cor-
ticoides) para evitar la complicación más importante que se puede producir que
son los aneurismas de las arterias coronarias.
Esta pregunta nos recuerda la pequeña cruz que llevamos a cuestas por el
hecho de haber elegido la medicina como medio de vida. Si nos encontramos
en la situación que dice la pregunta tenemos que ser capaces de reconocer ese 109. Señale cuál de las siguientes afirmaciones sobre la tosferina es correcta:
exantema como el que produce el virus de la varicela (lesiones cutáneas inicial-
mente máculas, posteriormente vesículas con contenido transparente que evo- 1. El agente causal es un coco gram positivo
lucionan a pústulas y luego a costras). Debemos también recordarle a la madre 2. Se produce generalmente en mayores de 1 año
que la enfermedad es contagiosa hasta que todas las lesiones estén en fase de 3. La fase de tos paroxística se produce durante una misma inspira-
costra. Entre las posibles respuestas figura la primoinfección herpética, que
ción
fundamentalmente afectaría a mucosas y como mucho a la piel peribucal, no
tendría lesiones por todo el cuerpo. Las lesiones típicas del síndrome de mano- 4. La clínica sugestiva en un ambiente epidemiológico puede ser sufi-
pie-boca se explicaran detenidamente en dermatología, fundamentalmente ciente para el diagnóstico
recuerda que las zonas de distribución son justo esas (mano, pie y boca). El 5. La causa más frecuente de fallecimiento por esta enfermedad son
impétigo infeccioso en una afectación más localizada en una determinada las complicaciones neurológicas
región, generalmente en la nariz o alrededor de la boca y en el enunciado sue-
len hablar de costras de aspecto melicérico, también se explicará con detalle en La pregunta se refiere a la tosferina, cuyo agente causal es un cocobacilo gram
dermatología. La erupción variceliforme de Kaposi tiene relación con ciertos negativo. Se produce generalmente en menores de un año, porque aun no han
virus (VHB) y se explicará en dermatología. sido vacunados. La cínica típica consiste crisis de tos paroxística en una misma
espiración, con un “gallo” final inspiratorio asociado a congestión facial, lagri-
106. Señale cuál de las siguientes afirmaciones sobre el eritema infeccioso es meo, protusión lingual, vómitos o pausas de apnea. La clínica sugestiva en un
INCORRECTA: ambiente epidemiológico puede ser suficiente para hacer el diagnóstico. La
causa más frecuente de fallecimiento por esta enfermedad son las complica-
ciones respiratorias (neumonías).
1. También es conocido como megaloeritema o 5ª enfermedad
2. El agente causal es el parvovirus B19
3. Se caracteriza por un exantema afebril que evoluciona en tres etapas 110. Sobre la parotiditis, señale cuál de las siguientes afirmaciones NO es
4. Puede causar artritis y artralgias correcta:
5. Una de sus complicaciones más graves es la aparición de una apla-
sia medular en enfermos con anemias ferropénicas graves 1. La infección está causada por un paramixovirus
2. La glándula afectada con más frecuencia es la parótida
Esta pregunta sirve para recordar que el eritema infeccioso también es conoci- 3. Puede aparecer como complicación una meningitis aséptica
do como megaloeritema o 5ª enfermedad. Su agente causal es el parvovirus 4. La orquitis en pospúberes causa con frecuencia esterilidad residual
B19 y se caracteriza por un exantema afebril que evoluciona en tres etapas (eri- total
tema en mejillas, exantema maculopapuloso y aclaramiento de las lesiones). 5. La parotiditis es la primera causa de sordera neurosensorial unila-
Una de las complicaciones más frecuentes es que puede causar artritis y artral- teral
gias y una de las más graves es la aparición de una aplasia medular en enfer-
mos con anemias hemolíticas graves. Esta pregunta es sobre la parotiditis, infección causada por un paramixovirus y
que generalmente afecta a la glándula parótida (aunque pueden verse afecta-
107. Uno de sus compañeros de carrera, internista actualmente, tuvo hace 8 das otras glándulas salivares). Entre las complicaciones de esta infección puede
meses un bebé y le consulta telefónicamente porque desde hace 3 días su hijo aparecer una meningitis aséptica, una orquitis en pospúberes (en la que es rara
presenta un cuadro febril sin ninguna otra sintomatología acompañante salvo la esterilidad residual total) y una sordera neurosensorial unilateral (la parotidi-
irritabilidad en los periodos de hipertermia. Esa misma mañana le ha aparecido tis es la primera causa adquirida de sordera nerviosa unilateral).
un exantema en tronco coincidente con la desaparición de la fiebre. ¿Cuál de
las siguientes afirmaciones le parece la más correcta? 111. Señale, sobre el VIH infantil, cuál de las siguientes afirmaciones le parece
INCORRECTA:
1. El diagnóstico más probable es una infección por sarampión
2. Seguramente el niño tiene también exudados amigdalares 1. La forma de transmisión más frecuente en menores de 13 años es
3. Estaría indicado un tratamiento con amoxicilina vía oral la vertical
4. La duración del exantema dura 4 semanas 2. El periodo de incubación oscila según el momento en el que se
5. Se asocia a infección por un virus herpético humano tipo 6 produce el contagio
3. La alteración crónica más frecuente es la neumonía intersticial lin-
Esta pregunta resume la clínica típica del exantema súbito, en el que lo típico foide
es que la fiebre desaparezca justo cuando aparece el exantema. Recuerda que 4. Las infecciones oportunistas son más frecuentes en los niños que

16 ] PEDIATRÍA [
www.academiamir.com Comentarios test de clase A Mir

en los adultos aunque su curso es más leve hace referencia a las contraindicaciones de la vacuna antipoliomielítica con
5. La neumonía por Pneumocistis carinii (o jiorveci) es la causa más virus atenuados (Sabin). Recuerda que este tipo de vacunas se deben evitar en
frecuente de mortalidad los inmunodeprimidos y en aquellos que convivan con estos (niños con SIDA,
inmunodeficiencias congénitas, en contacto con inmunodeprimidos o prema-
turos hospitalizados). De todas las opciones que nos dan, la única que no se
El siguiente bloque de preguntas se refiere al VIH infantil, nos detendremos
refiere a inmunodepresión es la que hace referencia a niños alimentados con
más en esta infección en la fase de consolidación, cuando ya se conozca lo sufi-
lactancia materna exclusiva.
ciente sobre la infección por VIH en el adulto. La forma de transmisión más fre-
cuente en menores de 13 años en nuestro medio es la vertical, es decir, los
niños se infectan porque se lo transmite su madre. El periodo de incubación 116. Un lactante sufre una intensa urticaria al administrarle huevo en el puré y
oscila según el momento en el que se produce el contagio, de manera que la las pruebas cutáneas y el RAST son positivas para ovoalbúmina, señale con cuá-
enfermedad es más grave y precoz si la infección se produce durante el emba- les de las siguientes vacunas debe tener precaución:
razo que si se produce postnatalmente. La alteración crónica más frecuente es
la neumonía intersticial linfoide. Las infecciones oportunistas son más graves, 1. Triple vírica (sarampión, rubéola y parotiditis)
pero menos frecuentes, en los niños que en los adultos y dentro de ellas la neu-
2. Tos ferina, tétanos y difteria
monía por Pneumocistis carinii (o jiorveci) es la causa más frecuente de morta-
lidad.
3. Hepatitis B
4. H. influenzae combinado con proteínas
112. En la analítica de un niño infectado por VIH se suele observar: 5. Poliomielitis tipo Sabin

Esta pregunta también hace referencia a las contraindicaciones de las vacunas


1. Linfocitosis
y también pertenece a un examen MIR anterior. Es una pregunta bastante
2. Inversión del cociente CD4/CD8 más precoz que en adultos directa y sólo sirve para recordar que la vacuna triple vírica (sarampión, rubéo-
3. Trombocitosis la y parotiditis) se cultiva en embriones de pollo y por tanto se debe evitar en
4. Hipergammaglobulinemia niños alérgicos a ovoalbúmina.
5. Respuesta aumentada de los linfocitos a mitógenos y antígenos
117. Acude a su consulta de pediatría un niño cuyo padre acaba de ser diag-
Esta pregunta hace referencia a los hallazgos analíticos típicos en los niños nosticado de tuberculosis pulmonar bacilífera. La prueba de la tuberculina en el
infectados por VIH. Lo habitual es encontrar linfopenia y trombopenia, la inver- niño es negativa, señale cuál sería la actitud correcta a seguir en este caso:
sión del cociente CD4/CD8 es más tardío en los niños que en los adultos, la res-
puesta de los linfocitos a mitógenos y antígenos está disminuida y suele existir
hipergammaglobulinemia.
1. Puesto que la prueba de la tuberculina ha resultado negativa, no
es necesario hacer nada más y se debe tranquilizar a la familia
113. Señale cual de los siguientes hijos de madre infectada por VIH está real- 2. Se debe iniciar profilaxis primaria con isoniazida durante dos meses
mente infectado: y repetir posteriormente la prueba
3. Se debe iniciar profilaxis secundaria con Isoniazida porque seguro
1. Niño de 6 meses seropositivo por ELISA que el niño está infectado y la prueba de la tuberculina no tiene valor
2. Niño de 3 meses seropositivo por Western-Blot en este caso
3. Niño de 10 meses que negativiza los anticuerpos maternos 4. No hacer nada por el momento y repetir el Mantoux en 10 semanas
4. Niño de 5 meses en situación inmunológica desconocida 5. Tratar al niño durante 6 meses con 3 fármacos (isoniazida, rifam-
5. Niño de 4 meses con PCR positiva para VIH picina y piracinamida)

Esta pregunta pertenece a un examen MIR y, aunque se insistirá en este tema


Esta pregunta hace referencia al diagnóstico de la infección por VIH. Como ya
en infecciosas, es importante recordar que la profilaxis tuberculosa en los niños
se ha mencionado previamente, los anticuerpos maternos (Ig G) pasan la pla-
es distinta de la de los adultos (dado que los fármacos son menos hepatotóxi-
centa y por lo tanto no sirven para hacer el diagnóstico de infección por VIH en
cos en los niños). Recuerda que la actitud depende de la edad del paciente y
los primeros meses de la vida porque no informan sobre el niño, sino sobre la
en la pregunta nos deberían mencionar la edad del niño. Como resumen ten
situación materna. El test del ELISA y el Western-Blot son técnicas que detec-
tan estos anticuerpos y por lo tanto no sirven para el diagnóstico en niños en cuenta el siguiente esquema:
menores de 18 meses. Si un niño negativiza los anticuerpos maternos tampo-
co está infectado, con el paso del tiempo ha eliminado los anticuerpos de la
madre y no ha fabricado los suyos propios. La respuesta 4 hace referencia a un Contacto en
niño cuya situación inmunológica es desconocida, luego no se le ha hecho nin- <15 años
guna prueba que indique que está infectado. La única respuesta que corres-
ponde a un niño infectado es la 5, ya que en ella no se miran los anticuerpos
sino si está o no presente el ARN del virus, y en este caso lo está.
PPD- PPD+
114. Un niño de 7 años es diagnosticado de meningitis por H.influenzae tipo b.
Señale cual sería la pauta más correcta de profilaxis entre los convivientes:

1. Vacuna conjugada anti-Hib a los menores de 5 años >6 años <6 años Rx tórax
2. Vacuna conjugada anti-Hib a todos
3. Rifampicina 4 días a todos
4. Cotrimoxazol 4 días a todos Isoniazida 2
5. Gammaglobulina específica anti-Hib a todos PPD en 2 meses meses + repetir Pato-
PPD Normal
lógica
Esta pregunta pertenece a un examen MIR y se refiere a un posible contacto
con un caso de meningitis por H.influenzae tipo b. La respuesta sería contro-
vertida porque la actitud más correcta en este caso sería la vacunación de todos Iniciar
- + Profilaxis
los niños menores de 5 años no vacunados o incompletamente vacunados (por
esto no es correcta la opción 1, que vacunaría a todos los menores de 5 años, 6-9 meses tratamiento
vacunados o no) junto con rifampicina durante 4 días. Si la pregunta vuelve a
caer y las opciones son las mismas, habría que contestar de nuevo la opción 3. Fin profi-
Rx tórax
laxis
115. Señale cuál de las siguientes circunstancias NO es una contraindicación
para administrar vacuna antipoliomielítica con virus atenuados (Sabin):
Pato-
1. Niños con SIDA Normal
lógica
2. Inmunodeficiencias congénitas
3. Niños alimentados con lactancia materna exclusiva
4. Niños en contacto con inmunodeprimidos Profilaxis Iniciar
5. Prematuros hospitalizados 6-9 meses tratamiento

Esta pregunta también pertenece a un examen MIR de años anteriores. En ella

17 ] PEDIATRÍA [
www.academiamir.com Comentarios test de clase A Mir

118. ¿Cuál de las siguientes afirmaciones sobre el síndrome de Münchausen por


poderes le parece INCORRECTA?

1. Habitualmente se trata de niños menores de 6 años


2. Muchas veces se fingen cuadros digestivos o abdominales
3. Suelen existir antecedentes de múltiples hospitalizaciones
4. Muchos casos padecen enfermedades osteoarticulares
5. Bastantes casos refieren manifestaciones hemorrágicas

Esta pregunta pertenece a un examen MIR y a un tema que no figura en nin-


guna parte del manual de pediatría. Sobre este tema se hablará en psiquiatría,
simplemente recuerda que cuando se menciona por poderes es porque es el
padre el que provoca el daño en el niño. Generalmente se trata de niños meno-
res de 6 años, ya que es más fácil fingir cuadros clínicos en niños que aun no
pueden contarlos síntomas por ellos mismos. Muchas veces se fingen cuadros
digestivos o abdominales y se refieren manifestaciones hemorrágicas (es fácil
añadir un poco de sangre de la madre o del padre a las heces o la orina del
niño). Lo habitual es que el padre o la madre que simula la enfermedad del
niño lo lleve a múltiples médicos por lo que suelen existir antecedentes de múl-
tiples hospitalizaciones. Lo que no es frecuente es fingir enfermedades osteo-
articulares (no es fácil convencer a un médico de una supuesta cojera si no la
objetiva en la exploración).

119. ¿Cuál de los siguientes fármacos es esencial en la parada cardiorrespira-


toria en pediatría?

1. Calcio
2. Dopamina
3. Noradrenalina
4. Atropina
5. Adrenalina

Las dos últimas preguntas del test pertenecen a exámenes MIR anteriores y
hacen referencia al tema de la reanimación cardiopulmonar en los niños.
Recuerda que la adrenalina es el fármaco más importante en la parada cardio-
rrespiratoria en pediatría.

120. Un lactante de 8 meses está jugando con una botella, consigue abrirla y
se mete el tapón en la boca con lo que produce una obstrucción aguda y total
de las vías aéreas superiores. La actitud más apropiada en este caso sería:

1. Intentar la extracción manual con el niño en decúbito prono y la


cabeza colocada a nivel inferior con respecto al tronco
2. Compresión abdominal brusca con reanimador colocado detrás
del paciente (maniobra de Heimlich)
3. Empleo repetitivo de golpes en la espalda y compresiones toráci-
cas con el paciente en Trendelenburg
4. Compresión abdominal brusca con paciente en decúbito supino y
cabeza por debajo del tronco
5. Broncoscopio urgente

Esta pregunta hace referencia a un episodio asfíctico en un lactante de 8


meses. Recuerda que al intentar la extracción manual del objeto podemos
empujarlo aun más dentro. La compresión abdominal brusca con reanimador
colocado detrás del paciente (maniobra de Heimlich) se debe emplear cuando
el paciente es más o menos de nuestra estatura y la compresión abdominal
brusca con paciente en decúbito supino y cabeza por debajo del tronco se
emplearía en niños que, sin ser de nuestro tamaño, son mayores que el niño
de la pregunta. Evidentemente sería de elección la extracción con broncosco-
pio urgente, que desgraciadamente no suele estar disponible siempre que nos
enfrentamos a una situación como la del caso clínico. La actitud más correcta
en este caso es el empleo repetitivo de golpes en la espalda y compresiones
torácicas con el paciente en Trendelenburg.

18 ] PEDIATRÍA [

Vous aimerez peut-être aussi